ENT EXAM MASTER eyes

¡Supera tus tareas y exámenes ahora con Quizwiz!

Case A 23-year-old woman presents to the emergency department due to double vision; pain upon moving the right eye; and numbness of the cheek, nose, and right side of the eye. She was playing baseball with her team and the ball hit her in the right eye. Upon physical exam, swelling, tenderness, numbness of the nose, and epistaxis are present Question What is the most likely diagnosis?

Correct answer: Blowout fracture Explanation The clinical picture is suggestive of blowout fracture, commonly known as an orbital floor fracture. The patient presents with a history of being hit in the right eye and presents with double vision, and numbness of the cheek and right side of eye. Additionally, X-ray would confirm a fracture. Blepharitis is not the correct answer. It is when oil glands become clogged or irritated. Additionally, a patient would present with itchy eyelids, watery eyes, a feeling that something is in their eye, or crust on the eyelashes. These symptoms are not present in this patient. Conjunctivitis is not the correct answer because conjunctivitis presents with burning, itching, irritation, and discharge, all of which is not present in this patient. Corneal abrasion is not the correct answer because it does not present with swelling, double vision, and numbness of the cheek. Instead, it is diagnosed with fluorescein stain and described as very painful, due to the scratch on the surface of the cornea. Stye is not the correct answer, as it is when bacteria gets into the oil gland in the eye lids. Additionally, it creates a red bump that is located closer to the eyelashes. It can make your eye feel watery, or as though something is in your eye. All of these symptoms are not present in the case.

Case Ico-delete Highlights An 18-year-old girl has had a small, slightly tender swelling in her left upper eyelid for 10 days. For the first day or two, it was red and a little painful. Now it is painless, although it has grown in size. There has not been any drainage, visual changes, or itching noted. She has been well otherwise. On exam, vitals are normal, extraocular muscles are intact, and pupils are equal and reactive to light. Her left upper eyelid has a 1.5 cm round, nontender, swollen mass that is mildly erythematous without any drainage. The underside of the lid is grayish-red. Question Ico-delete Highlights Based on these findings, what is the most likely diagnosis?

Correct answer: Chalazion Explanation A chalazion is an enlargement of a deep oil gland in the eyelid that results from an obstruction of the gland opening at the edge of the eyelid. Associated factors include immunodeficiency, leishmaniasis, seborrhea, high blood lipid concentrations, acne rosacea, tuberculosis, chronic blepharitis, carcinoma, and viral infections. Most will disappear without treatment after 1 to 3 months. If hot compresses are applied several times a day, it may disappear sooner. Persistent lesions that cause changes in vision should be examined by an ophthalmologist for possible drainage by incision and curettage or intralesional corticosteroid injection. Hordeolum, or stye, is an acute infection of one or more of the glands along the edge of the eyelid or under it. This is usually caused by a staphylococcal infection. An abscess may form and rupture. A stye will usually last 2 - 4 days and may form simultaneously with or as a result of blepharitis. It begins with redness, tenderness, and pain at the edge of the eyelid. A small round, tender swollen area often forms with a tiny, yellowish central spot. Blepharitis is an inflammation along the edges of the eyelids with possible thickening scales, crusts, shallow ulcers, or inflamed oil glands. This may be caused by staphylococcal infection of the eyelids or the ducts of deeper glands, seborrheic dermatitis of the face and scalp, or acne rosacea. The eyes and lids may itch and burn, or they may become watery and photosensitive. The eyelid may swell and become red; lashes may fall out. Sometimes small abscesses containing pus can develop in the sacs at the base of the eyelashes and form shallow ulcers known as ulcerative blepharitis. Crusts may form on the edges of the eyelid, and when removed, they may leave a bleeding surface. This tends to recur and resist treatment, but the crusts rarely damage the cornea or causes vision loss. Dacryostenosis results in a blockage of the flow of tears from the eye to the nose due to narrowing of the nasolacrimal ducts. This can result from inadequate development of any part of the ducts, chronic nasal infection, severe or recurring eye infections, or fractures of the nasal or facial bones. If dacryostenosis does not resolve, infection or dacryocystitis will result. Dacryocystitis is an infection of the lacrimal sac that results from a blockage of the nasolacrimal duct, which leads from the lacrimal sac into the nose. This may occur acutely or chronically. In acute infections, the area around the lacrimal sac becomes painful, red, and swollen; this causes the eye to become red and watery as well as ooze a purulent discharge. Pressure to the lacrimal sac may push pus through the opening near the nose. Fever is common. Infection can sometimes cause fluid to be retained in the lacrimal sac, which becomes a mucocele that can become an abscess.

Case A 42-year-old man presents with a firm, painless bump on his left upper eyelid. On examination, you note a 5 mm mass within the tarsus of the left eye. The skin is freely movable over the mass. The remainder of the eye exam is unremarkable. Question What is most likely diagnosis?

Correct answer: Chalazion Explanation The clinical picture is an example of a chalazion. A chalazion is a painless chronic mass in the eyelid. Chalazions differ from hordeolums in that they (chalazions) are usually painless, apart from the tenderness caused when they swell up, and are generally larger in size than styes. Ectropion is when the eyelid sags outwardly and the lid does not close well. Hordeolum are acute, red, and painful. Pterygium involves the sclera.

Question Which of the following drugs used in glaucoma therapy has a prolonged longest duration of action and can be administered relatively infrequently?

Correct answer: Echothiophate Explanation Elevated intraocular pressure, if untreated, can lead to retinal and optic nerve damage, restriction of the visual fields, and blindness. The open-angle form of glaucoma is a chronic condition involving elevated intraocular pressure that often responds to drug therapy. A variety of topical drops are available for direct instillation into the conjunctival sac for treatment of this condition. Intraocular pressure is a function of the volume of aqueous humor present in the anterior cavity of the eye. The production of aqueous humor by the ciliary epithelium is normally balanced by the drainage of aqueous humor by the trabecular system at the anterior chamber angle. Ciliary epithelium possesses β-adrenergic receptors that are coupled with the cyclic adenosine-3',5'-monophosphate (cAMP) secondary messenger system. Activation of this cyclic nucleotide system by beta-adrenergic stimulants causes the ciliary epithelium to increase the output of aqueous humor. Therefore, beta agonists such as albuterol can worsen the elevated intraocular pressure of glaucoma. Muscarinic antagonists such as atropine, ipratropium, and tropicamide produce mydriasis and cycloplegia. The blockade of ciliary muscle function by antimuscarinics decreases the anterior chamber angle and impedes the drainage of aqueous humor, thus exacerbating a condition of glaucoma. Drug treatment of glaucoma is directed at reduction of aqueous humor secretion, improvement of aqueous drainage, or both. Beta-adrenergic blockers, diuretics, cholinomimetics, and alpha-adrenergic agonists are used in glaucoma therapy: Reduction of aqueous humor secretion Beta blockers are front-line drugs employed in glaucoma therapy. These derivatives of propranolol are effective in open-angle glaucoma and offer dosing convenience while producing relatively few adverse effects. The ophthalmologic drugs are available as topical drops and include timolol, betaxolol, carteolol, metipranolol, and levobunolol. Diuretics such as the carbonic anhydrase inhibitor, acetazolamide, reduce the availability of bicarbonate ion (HCO31-), a critical component of aqueous humor. As a result the secretion of aqueous humor by the ciliary epithelium is diminished. Enhancement of aqueous humor drainage Cholinomimetics are front-line drugs in the treatment of open-angle glaucoma. The cholinomimetics (or cholinergic agonists) include direct-acting choline esters with M3-muscarinic activity such as pilocarpine and carbachol, and indirect-acting cholinesterase inhibitors such as physostigmine and echothiophate (an ultra-long-acting organophosphate). Cholinesterase inhibitors are nonspecific cholinergic agonists that bind available cholinesterase enzymes such as acetylcholinesterase and plasma cholinesterase. Compared to acetylcholine, these drugs are more slowly hydrolyzed from the enzyme. More importantly, cholinesterase is prevented from inactivating acetylcholine while cholinesterase inhibitor molecules are attached. Physostigmine exhibits reversible cholinesterase inhibition whereas anticholinesterases such as the organophosphates, echothiophate and isoflurophate, produce extended action in glaucoma therapy. The prolonged duration of effect of these cholinergic drugs aids in patient compliance in the treatment of glaucoma. The extended effect is due to the irreversible nature of the cholinesterase enzyme inhibition and the time required to synthesize new enzyme to replace that which has been hydrolyzed. (It should be noted that the risk of cataract development is higher with the organophosphate derivatives than with other drugs used in glaucoma therapy.) The cholinomimetics produce miosis and contraction of the ciliary muscle which increases tension on the trabecular meshwork, opens pores, and improves the drainage of aqueous humor into the canal of Schlemm. Alpha agonists such as epinephrine cause an increased outflow of aqueous humor through the trabecular meshwork at the anterior chamber angle.

Case During your newborn nursery rounds, a young new mother tells you that there is a family history of eye problems that run on her dad's side. She's not sure what problem it is exactly, but many relatives have had to wear glasses. On exam, the infant's eyes seem to be deviated toward the nose. Corneal light reflex testing confirms your suspected diagnosis. Question Which of the following conditions is most likely present?

Correct answer: Esotropia Explanation The most likely diagnosis is congenital esotropia. This condition causes the eyes to deviate toward the nose, giving the appearance that they are crossed. The corneal light reflex test, also called the Hirschberg test, can reliably diagnose the condition. The corneal light reflex or small white dot on the front of the eye should be in the same position in each pupil. It is normally just slightly nasal to the center of each pupil. If the position is different in each pupil, then some type of strabismus is present. A prism diopter (PD) is the unit measuring the deflection of light passing through a prism equal to a deflection of 1 cm at a distance or 1 meter. Infantile esotropia characteristically presents as a constant, moderate-to-large angle measuring approximately 25 to 60 PD with alternate fixation. Infants presenting at 2 to 4 months of age with constant esotropia of 40 PD or greater are valid candidates for surgical repair. Cataracts present as an opacification or clouding of the lens. Many cataracts in children are actually acquired within the first several years of life. A normal and equal red reflex in each eye will exclude cataracts. This is best done with the room lights dimmed and an ophthalmoscope held about 2-3 feet away from the infant so both pupils can be seen simultaneously, making comparisons easier. Dark lens opacities signify possible cataracts that will eventually produce a gray or white pupillary reflex, or leukocoria. It is then best evaluated by a slit-lamp after pupillary dilation. Glaucoma is a common, progressive disease characterized by elevated intraocular pressure causing progressive damage to the optic nerve that results in atrophy and blindness. This is due to improper development of the eye's aqueous outflow system. Diffuse corneal haze will obscure the pupil and iris markings; in addition, the symptomatic triad of epiphora or excessive tearing, photophobia, and blepharospasm is evident. The corneal light reflex is normal in glaucoma. Retinoblastoma usually presents with leukocoria and strabismus. It can also be detected with an abnormal red reflex test. In the absence of a white pupil, this diagnosis is unlikely in the case presented. Dacryocystitis is an inflammation of bacterial infection of the lacrimal sac. It will present with swelling and erythema of the inner portion of the eye, near the tear duct

Case A 12-year-old boy presents with fatigue and jaundice. History obtained from the patient and his mother is negative for recent illness, fever, infectious exposures, medication, alcohol, or drug use. He denies gastrointestinal (GI) symptoms and a history of GI disease. On physical examination, he appears ill; the liver edge is palpable and slightly tender. Skin and sclera are icteric, and there is corneal discoloration. On eye examination using a slit-lamp, you note brown-yellow rings encircling the iris in the rim of the cornea bilaterally. You order a serum ceruloplasmin level, which is reported as low. Question What is this diagnostic corneal pigmentation known as?

Correct answer: Kayser-Fleischer rings Explanation Kayser-Fleischer rings are the result of the accumulation of copper in the cornea; they are the most unique sign of Wilson's disease. Wilson's disease is an inherited disorder of copper toxicity due to a genetic defect in copper transport. Beginning at birth, copper is not secreted into the bile or incorporated into the copper protein ceruloplasmin, resulting in low serum levels of ceruloplasmin. Symptoms and signs develop age 5-40 as copper accumulates in the liver, brain, cornea, kidney, and reproductive organs. 50% of patients present with hepatitis, 40% present with neurological manifestations (tremor, speech disorders, dysphagia, incoordination), and 5-10% first present with Kayser-Fleischer rings, amenorrhea, miscarriages, or hematuria. Diagnosis is confirmed by Kayser-Fleischer rings on slit lamp examination in the presence of a low serum ceruloplasmin. AST and ALT levels are often elevated; serum copper is low; 24-hour urinary copper excretion is elevated. Treatment is lifelong chelation or oral zinc and a low copper diet. Keratoconus is a bulging of the cornea to form a cone, and the classic sign is Fleischer's rings, which are iron-colored rings surrounding the cone. This progressive bulge is due to a weakness in the cornea and often occurs bilaterally at age 10-20. There are frequent changes in visual acuity, necessitating repeated prescription changes; contacts provide better correction than glasses. Corneal transplant may be necessary if corrective lenses are not adequate. Arcus juvenilis is a gray or white arc around the peripheral cornea similar to arcus senilis in adults. It occurs in younger adults and is often associated with high blood cholesterol. A metallic foreign body lodged in the cornea can quickly result in a single small-diameter rust ring that requires ophthalmologic intervention with a rust ring drill for removal. A pinguecula is a raised, yellowish discoloration on the bulbar conjunctiva at the 3 o'clock or 9 o'clock position of the scleral-corneal junction. It is a benign growth caused by an accumulation of conjunctival tissue that can be the result of chronic actinic irritation.

Case A 26-year-old man presents with an eye issue. He does not wear corrective lenses. The only change of lifestyle that he reports is that a few months ago, he quit his office job and began to "help out a buddy" in the construction business. On physical examination, there is a triangular fold of tissue extending from the medial conjunctiva to the cornea in both eyes. Question What is the most likely diagnosis?

Correct answer: Pterygium Explanation The clinical picture is suggestive of pterygium. The common presentation is a fleshy, triangular intrusion of the conjunctiva onto the nasal side of the cornea that is often bilateral. It is usually associated with constant exposure to sand, wind, or sunlight. Keratoconjunctivitis sicca (or dry eyes) is a condition of lacrimal gland hypofunction commonly seen in older women. Pinguecula is a yellow elevated conjunctiva nodule in the area of the palpebral fissure on the nasal side of the eye. It is common in patients over 35. Chalazion is a chronic sty. Hordeolum is an acute sty.

Case A 23-year-old man presents 2 hours after being involved in a road traffic accident in which he sustained right-sided periorbital injuries. He is seeing double; he denies headache, vomiting, and loss of consciousness. On examination, he is alert and oriented to time, space, situation, and person. His right eye deviates downwards and temporally. Question What other finding would you expect to be present in this patient?

Correct answer: Ptosis Explanation The correct response is ptosis due to denervation of the levator palpebrae superioris, which lifts the eyelid. The clinical picture is suggestive of injury to the oculomotor nerve, which is the third cranial nerve. It innervates the following muscles: Superior rectus Inferior rectus Medial rectus Inferior oblique Levator palpebrae superioris Pupilloconstrictor Corneal anesthesia with loss of corneal reflex is a result of interruption of the trigeminal nerve supply to the cornea and conjunctiva. It does not involve the oculomotor nerve. Denervation of the pupilloconstrictor causes mydriasis, not miosis. Because of the uninhibited function of the lateral rectus, denervation of the oculomotor nerve results in abduction of the eye. Causes of third cranial nerve palsy include: Intracranial and intraorbital lesions (e.g., neoplasms) Head and orbital trauma Ocular myopathies Cerebral aneurysms Transtentorial herniation

A 62-year-old woman presents to the emergency department with acute unilateral loss of vision for 1 hour. Fundoscopic examination demonstrates vein dilation, intraretinal hemorrhages, and cotton-wool spots with optic disc swelling. What is the most likely diagnosis?

Correct answer: Retinal vein occlusion Explanation Retinal vein occlusion results in acute vision loss with retinal vein dilation, intraretinal hemorrhages, cotton-wool spots, and optic disc swelling on fundus examination. Macular degeneration does not cause acute vision loss but progressive loss over time and does not result in the findings above. Retinal artery occlusion does not cause retinal vein dilation and causes a classic cherry-red spot at the fovea. Diabetic retinopathy does not cause the constellation of signs noted above. Retinal detachment was not found on fundoscopic examination.

Case A 10-day-old male infant presents with bilateral conjunctivitis with moderate white discharge. He is acting normally, has no fever, and is feeding well. He was born full term without any complications. His mother had minimal prenatal care. He has been gaining weight well. On exam, he is alert and active. Culture with immunofluorescence reveals inclusion bodies. Question What is the most likely cause of this infant's conjunctivitis?

Correct answer: Chlamydia trachomatis Explanation Chlamydia and gonorrhea are the most common causes of conjunctivitis in the neonate. Given the timing and presentation, Chlamydia is the most likely cause of this infant's conjunctivitis. A characteristic finding of chlamydial infection is the presence of inclusion bodies in the epithelial cells of a conjunctival smear. The usual incubation period for C. trachomatis is 5-14 days and 2-5 days for N. gonorrhoeae. Gonococcal conjunctivitis tends to produce a more purulent discharge compared to C. trachomatis. Herpes conjunctivitis is sometimes contracted when infants are born to mothers with herpes lesions in the genital region, which is not the scenario in this case. Conjunctivitis due to silver nitrate drops usually occurs 6-12 hours after birth. This is not the presentation of this infant. Coxsackievirus is not a common cause of conjunctivitis in neonates.

Case You are evaluating a 24-year-old woman for bilateral eye pain. She describes red, itching, irritated eyelids for several weeks. She states she has had "several bouts" of similar symptoms over the last few years. Exam is consistent with blepharitis. She does not wear contacts, and she occasionally wears eye makeup. She denies any other infectious complaints. Question What is the most appropriate treatment?

Correct answer: Discuss good eye hygiene and eyelid scrubbing Explanation The correct answer is that you should discuss good eye hygiene and eyelid scrubbing with the patient. Blepharitis is chronic inflammation of the eyelids. Patients often present with irritation of the lid margins that may produce minor crusting, a gritty sensation when blinking, eye itching and redness but a full preservation of vision. The majority of patients have recurrent bouts of inflammation, which can be resolved by eyelid washing and avoidance of eye makeup.1 The treatment of blepharitis rarely requires antibiotics. While it is possible to have a secondary infection, typically staph aureus, there is no evidence of a secondary infection in our patient.2 Initiation of ophthalmic steroids should be done under the guidance of an ophthalmologist. Additionally, there is no benefit of an ophthalmic steroid in the treatment of blepharitis. The majority of cases of uncomplicated blepharitis do not need to be referred to an ophthalmologist unless the diagnosis is uncertain or there are other complicating factors. Additionally, there is no surgical correction for uncomplicated blepharitis. Our patient does not have signs and symptoms of increased intraocular pressure.

Question A 16-year-old boy was at his team basketball practice when a team member forcefully jabbed his fingers into the 16-year-old's left eye while trying to block his shot. He felt sharp, blinding pain and has had much tearing. He was taken out of practice, ice applied, and sent to you for evaluation. Which examination should always be performed initially following direct eye trauma?

Correct answer: Evaluate visual acuity with corrected vision in place Explanation The initial goal of management is to provide timely recognition of injury to allow for management and stabilization of the condition. On the sidelines, this should always begin with an evaluation of visual acuity with corrected vision in place, while taking a thorough history. A deficit in visual acuity unexplained by a previous abnormality or refractive error (astigmatism, hyperopia, myopia, or presbyopia) strongly suggests a serious globe injury. This can be done by having the player read words or letters or count fingers held at a 2-5 foot distance. Painful maneuvers and palpation should be left to the end, such as examining surrounding structures of the eyes, including the orbital bones, eyelids, and medial canthus. Assess the lacrimal drainage system for swelling or lacerations. Topical anesthesia may be required with severe pain or photophobia. The usual mechanism of eye injury is direct trauma. When the eye is struck by an object that is smaller than the orbit, the globe may become compressed. Objects larger than the orbital opening directs force through the skull to the brain that may result in optic nerve, cerebral, or bony injury. Other injuries following optical trauma include corneal abrasions, traumatic rupture of the globe, hyphema or hemorrhage into the anterior chamber of the eye, introduction of a foreign body, lacerations to the eye and surrounding structures, dislocation of the intraocular lens, retinal detachment, and orbital "blow-out" fractures. After transfer to a fully equipped facility, evaluation of intraocular pressure, anterior adnexal, and the interior eye by means of biomicroscopy and ophthalmoscopy should be done. After assessing visual acuity, surrounding structures may be examined, including the globe and sclera, for hyphema as well as corneal defects and lacerations. These findings might further suggest globe injury. Extraocular movements should also be observed for symmetry and nystagmus by having the patient follow a finger in all visual fields. Checking the pupil for size, shape, symmetry, and reaction to light should also be done after visual acuity testing. Sensation over the cheek checks for infraorbital nerve injury that would suggest a blow-out fracture.

Case A 33-year-old man presents with a 1-day history of a painful left upper eyelid. He denies any change in vision, discharge, trauma, or foreign body. The pain started after the patient was cleaning out the garage. On physical exam, the visual acuity is OD/OS/OU = 20/20. The lateral aspect of the left upper eyelid is swollen, erythematous, and tender to palpation. The rest of the eye exam is normal. Question What is the most likely diagnosis?

Correct answer: Hordeolum Explanation A hordeolum is an acute localized red, swollen, and tender area on the upper or lower eyelid. A chalazion is a chronic granulomatous inflammation of a meibomian gland that is characterized by a hard, nontender swelling of the upper or lower eyelid. It is usually not erythematous. Blepharitis is a chronic bilateral inflammation condition of the eyelids. Dacryocystitis is an infection of the lacrimal sac located in the nasolacrimal area. Uveitis is an intraocular inflammation characterized by photophobia, blurred vision, and moderate pain in the eye.

Case A 33-year-old man presents with acute left eye pain. He was working in his garage on a woodworking project, and as he hammered in a nail, he felt that something hit him in the left eye. On examination, you note that the left pupil has a teardrop appearance. Question What is the next best step in the management of this patient?

Correct answer: Ophthalmology referral Explanation The clinical picture is suggestive of an intraocular foreign body or penetrating injury to the eye. This is commonly seen in individuals with a history of pounding on metal or using grinding equipment. The patient may give a history of "something hitting my eye" or "something was pulled out of my eye." His pupil is teardrop shaped, indicating penetration of the globe. Patients with suspected intraocular foreign body must be referred emergently to an ophthalmologist. Fluorescein staining is indicated for corneal abrasions/foreign bodies. To avoid extrusion of intraocular contents, EOMs should not be performed. Visual acuity should be tested, but alterations in visual acuity will not confirm your diagnosis. Testing intraocular pressure is indicated if you suspect glaucoma.

Case Ico-delete Highlights A 1.5-year-old boy presents with a squint in the left eye. His mother informed you that the child's eyes were quite normal until about 2 months ago when she noticed asymmetric movements of her son's eyes. She also felt that the child could not see properly with his left eye. There is no history of trauma to the eye. The child was born at full term and his growth and development have been within normal limits. On exam, the pupils are equal in size. There is loss of vision in the left eye and a convergent squint in the same eye. Fundus examination showed absence of red reflex in the left eye, and instead a white pupillary reflex (leukocoria) was seen. X-ray of the skull shows calcification within the globe. Question Ico-delete Highlights What is the most likely diagnosis?

Correct answer: Retinoblastoma Explanation The most likely diagnosis is retinoblastoma, as it is the most common primary ocular tumor in children below 5 years of age. 90% of cases are diagnosed below 3 - 4 years of age. The index case is a 1.5-year-old boy who has presented with a recent appearance of squint and absence of normal red reflex in the left eye, replaced instead by a white pupillary reflex (leukocoria). This is due to reflection of light from the white-colored tumor and loss of vision in that eye. The diagnosis is further supported by calcification seen in the globe in the X-ray of the skull. Fundoscopy may show the tumor as a white mass, which may be small and flat or may be large and protuberant. Orbital inflammation, hyphema, and irregular pupil are seen in advanced stages of the disease. Retinoblastoma gene is a recessive gene located on the chromosome13 at the 13q 14 regions, and the tumor may arise from any of the nucleated layers of the retina. Retinal detachment in infants and children more commonly occurs due to trauma, secondary to other abnormalities like myopia, or after cataract surgery. It can also occur in diabetes, sickle cell disease, and retinopathy of prematurity. Presenting signs can be loss of vision, secondary strabismus (squint), nystagmus, and leukocoria (white pupillary reflex). Calcification seen on an X-ray of the skull in retinoblastoma is absent in retinal detachment. Also ultrasonography and neuroimaging may be required to establish the cause of detachment. Congenital glaucoma (elevated intraocular pressure) usually manifests during the first 3 years of life. The classical triad of symptoms of congenital glaucoma are epiphora (excessive lachrymation), photophobia (sensitivity to light), and blepharospasm (squeezing of the eyelids). These symptoms are due to corneal irritation. As the cornea and sclera are more elastic during early childhood, the elevated intraocular pressure therefore leads to expansion of the eyeball, including the cornea, and development of buphthalmos (ox eye), which means a large eye. This leads to corneal edema and conjunctival congestion. The cornea may become cloudy. There is no white pupillary reflex or calcification in the globe seen on an X-ray of the skull. A cataract is an opacity in the lens and may cause significant impairment of vision. It may be an isolated defect or may be a part of a generalized disorder. Common causes are intra uterine infections like rubella, cytomegalovirus infection, toxoplasmosis, metabolic disorders like galactosemia, and chromosomal disorders like trisomy 13, 18, and 21. Trauma to the eyeball is a major cause of cataract in children. The red reflex may be absent or may be irregular or there may be a white pupillary reflex. The retina and the blood vessels may not be visualized due to the lenticular opacities. Nystagmus may be present. Poor visual fixation, squint, and poor social smile may be seen later on. Calcification in the globe is not present in cataract. Persistent hyperplastic primary vitreous (PHPV) is caused by persistence of portions of the fetal hyloid vascular system and the associated fibromuscular tissue. The condition is usually unilateral, and the affected eye is smaller than normal. The anterior chamber is shallow, and the lens is also smaller than normal. Other presenting signs are white pupillary reflex (leukocoria) strabismus and nystagmus. The course is progressive and outcome is poor.

Case A 6-year-old girl presents with a 2-day history of a small, tender, superficial abscess on her left upper eyelid at the lid margin. She denies discharge, fever, or trauma. There is mild pain on palpation, and she has a normal ophthalmic exam. Question What treatment should be given?

Correct answer: Warm compresses Explanation This description is of a hordeolum, which is also called a stye. The usual causative agent is Staphylococcus aureus. Treatment involves warm compresses and frequently a topical ophthalmic antibiotic. Occasionally, surgical incision and drainage are required. There is no need for a systemic antibiotic for a minor infection, and the causative agent does not appear to be allergic or viral. A stye can be confused with a chalazion, which is a granulomatous inflammation of a meibomian gland characterized by a nontender nodule. The lesion tends to be chronic, and it shows no sign of acute inflammation

Case An 18-year-old man presents with blurred vision and some eye pain that began 2 days ago and has become progressively worse. Upon examination, the eye is slightly edematous with a white to yellow exudate present under the eyelid and at the corner. The rest of his clinical and physical history is unremarkable. A conjunctival scraping is obtained and gram stained. Based on the gram stain result, the conjunctival scraping was sent to the laboratory for culture and sensitivity. The patient is given instructions for topical antibiotic ointment treatment (polymixin B/trimethoprim) to be administered every 2-4 hours for 7-10 days. Pathology later shows that the conjunctival scraping culture grew out a beta hemolytic organism that was catalase positive, coagulase positive, and gram stained as gram-positive cocci. Question Ico-delete Highlights What is the most likely causative organism of the patient's conjunctivitis?

orrect answer: Staphylococcus aureus Explanation Staphylococcus aureus is a gram-positive staining cocci that is catalase positive, coagulase positive, and frequently beta hemolytic on blood agar. It is probably the 2nd most common bacterial isolate of human infections behind Escherichia coli and the most common cause of bacterial endophthalmitis. The organism has been described as an etiologic agent of many infections, including but not limited to, conjunctivitis, endocarditis, septicemia, abscesses, and urinary tract infections. The conjunctivitis caused by Staphylococcus aureusis usually characterized as non-severe, where there is little to no lid edema, scant purulent discharge, and normal cornea; however, in some cases the presentation can be severe. Chlamydia trachomatis is an obligate intracellular parasite with a unique biphasic life cycle. It does not gram stain, and laboratory procedures used for diagnosis include isolation in tissue culture, EIA detection of antigen, immunofluorescent staining, cytologic examination for intracytoplasmic inclusions, and by the demonstration of nucleic acid by direct hybridization or by amplification techniques. It can cause inclusion conjunctivitis and ocular trachoma. The inclusion conjunctivitis presents as an acute follicular conjunctivitis and is usually self-inoculated from an infected genitourinary site. The patient frequently complains of a foreign body presence in the eye. These symptoms are usually unilateral, and in the first 2 weeks, there is a mucoid discharge that becomes purulent. Pseudomonas aeruginosa is a gram-negative rod, non-lactose fermenting, oxidase-positive motile bacteria. Pathogenesis is due to its minimal nutritional requirements, relative resistance to antibiotics, and a host of other invasive and toxinogenic substances that it produces. It can cause a keratitis that is rapid in its development. The infection is usually the result of a previous injury to the eye, which causes an interruption in the epithelial surface and allows bacterial invasion of the underlying stroma. Scrapings from the floor of the ulcer exhibiting gram-negative rods are strongly indicative of Pseudomonas aeruginosa and should necessitate treatment. Haemophilus aegyptius is a gram-negative coccobacillus, non-motile, fastidious bacteria requiring the presence of special factors for its growth on agar media. These factors are hemin and nicotinamide adenine dinucleotide, which are present in chocolate agar but not on other isolation media. The organism is indigenous to humans. It is an important cause of a purulent conjunctivitis called 'pink eye' and can occur in outbreaks because of its contagious nature. The diffuse pink color of the sclera and the presence of a serous or purulent discharge are virtually diagnostic of Haemophilus aegyptius infection. Leukocytosis is absent. The infection is not acute in presentation. Bacillus cereus is a gram-positive (or gram-variable) rod that is aerobic, spore-forming, and is ubiquitous in nature. Bacillus cereus is an important cause of food poisoning. It has also been recognized as an ocular pathogen. The ocular infection is acute in presentation and requires aggressive intervention to save the eye. The presence of progressive corneal deterioration and ring abscess formation is a complication of panophthalmitis caused by Bacillus cereus. Except for infections with Pseudomonas aeruginosa, this finding is almost pathognomonic of Bacillus cereus. Because of the seriousness of the infection, early diagnosis is important. Patients presenting with ocular infections after trauma or in the setting of drug abuse should arouse suspicion. Acanthamoeba is a free-living amebae that can cause granulomatous amebic encephalitis and keratitis. It can not be cultured by routine culture methods. Detection is usually made by observing the free living motile organisms in a wet prep preparation. Acanthamoeba keratitis is a slow-developing corneal infection that occurs in healthy people and is usually associated with contact lens wearers. Symptoms include blurred vision, conjunctivitis, tearing, severe pain to the eye, and photophobia. The keratitis achieves an advanced stage in several days to several months and can exhibit patchy stromal infiltrates and dendriform epithelial involvement without frank corneal ulceration in its early stages. Candida albicans is a yeast. Yeasts appear on gram stain as large gram-positive organisms, approximately 3-5 times larger than gram-positive cocci, and are nonhemolytic on blood agar. They are aerobic and generally grow well on most non-selective agar media. Endophthalmitis due to yeast is generally a common and serious complication of intravenous drug use. Candida albicans is the most common fungal cause. It is usually of hematogenous origin where the patient has infective endocarditis or some other infective process occurring. The symptoms are blurred vision, decreased vision, white cotton appearing exudative lesions in the choroid and retina with vitreous haziness, and eye pain. A definitive diagnosis is made by obtaining vitreous fluid for gram stain and culture.

Case A 28-year-old man presents with diplopia and the inability to move the right eye outwards. He was hit by a ball on the right side of his face while playing volleyball 2 hours ago. His symptoms are non-progressive. On examination, his visual acuity is normal in both eyes. Right eye is medially deviated and cannot be moved laterally; otherwise, there is no abnormality detected. Question What nerve is most likely injured?

Correct answer: Abducens Explanation The abducens, or cranial nerve VI, is the most common nerve traumatized in head injuries. It is the only supply to the lateral rectus muscle of the eye, which is responsible for external or lateral deviation of the eye. Injury to this nerve results in esotropia (internal deviation) of the eye and failure to move the eye outwards. The oculomotor, or cranial nerve III, supplies most extraocular muscles except the SO (superior oblique) and lateral rectus. Injury results in external deviation, ptosis, and pupil dilatation. The trochlear nerve supplies the SO. An injury to the nerve alone is rare, resulting in defective downward and inward gaze. The facial, or cranial nerve VII, supplies the facial muscles; an injury to the nerve results in facial paralysis (i.e., inability to close the eye, inability to frown with the forehead, and loss of mouth movement with smiling). The trigeminal nerve supplies the muscles of mastication and sensation of the face. An injury to this nerve results in defective facial sensation and mastication.

Case A 64-year-old Asian man presents with a 1-hour history of severe right eye pain that started while he was watching a movie at the theater. He notes blurred vision and seeing halos around lights when using his right eye. He denies loss of vision, trauma, discharge, and any symptoms in the left eye. His last eye exam was 6 months ago, which resulted in new glasses. Past medical history is negative, and the patient denies any allergies. On physical exam, visual acuity is OS 20/25, OD 20/70, and OU 20/40. Pupil on right eye is 7 mm and left eye is 3 mm. Right pupil is non-reactive to light; left pupil is reactive to light. Right cornea is steamy in appearance and left cornea is clear. Question Ico-delete Highlights What is the most likely diagnosis?

Correct answer: Acute angle-closure glaucoma Explanation Acute angle-closure glaucoma is frequent among the older age group and in the Asian population. Essential for diagnosis is rapid onset of severe pain and profound vision loss/blurring with halos around lights. On physical exam, a fixed and dilated pupil and a red eye with a steamy cornea are the hallmarks. Chronic glaucoma is usually an insidious onset of bilateral loss of peripheral vision. Cataracts usually present as gradually progressive and non-painful; they present with clear corneas, normal pupil size, and normal light reaction. Acute uveitis presents with blurred vision; the cornea is clear and the affected pupil is small with poor papillary light response. Acute conjunctivitis presents with copious discharge; it does not impact vision. There is a clear cornea, normal pupil size, and normal papillary light reaction.

Case A 40-year-old man presents with severe pain in his left eye, decreased vision, nausea, and abdominal pain. On examination, the patient's left pupil is moderately dilated and nonreactive. The cornea is 'steamy' in appearance and generally the eye is red. Question What is the diagnosis?

Correct answer: Acute angle-closure glaucoma Explanation This patient has acute angle-closure glaucoma. The typical characteristics of this condition are all exhibited by this patient (i.e., steamy cornea, severe pain, blurred vision, dilated and nonreactive pupil). A physical exam finding in a patient with acute uveitis would be a small pupil. In addition to these symptoms, patients may also present with abdominal pain. A corneal infection and corneal ulcer would cause circumcorneal injection and watery, or purulent, discharge.

Question A previously healthy 5-year-old boy presents with complaints of bilateral eye pain and redness. The child's mother reports that several children in his daycare facility have had similar symptoms. Results of physical exam indicate bilateral nonsuppurative conjunctival inflammation, photophobia, and preauricular lymphadenopathy. What is the most likely organism responsible for this outbreak of conjunctivitis? 1 Haemophilus ducreyi 2 Adenovirus 3 Neisseria gonorrhoeae 4 Chlamydia trachomatis 5 Human herpesvirus 6

Correct answer: Adenovirus Explanation A common source outbreak of epidemic keratoconjunctivitis, 'pink eye', is usually a result of contact with other infected individuals or improperly cleaned optical instruments. In this case, the acute onset, bilateral presentation, and epidemic nature point to adenovirus (types 8, 19, and 37) as the most likely pathogen. Ophthalmic infection with this highly communicable virus, for which humans are the only known reservoir, is about the only time the preauricular lymph node is evident. Haemophilus ducreyi is the causative agent of chancroid, a sexually transmitted disease causing a hard, localized chancre in the genital area. Neisseria gonorrhoeae may cause conjunctivitis in adults and can be transmitted to neonates. Chlamydia trachomatis is the leading cause of blindness worldwide. Infection with this obligate, intracellular pathogen results in trachoma (a chronic keratoconjunctivitis) in endemic areas such as Africa, Asia, and the Mediterranean. Chlamydia trachomatis also causes inclusion conjunctivitis, the most commonly seen infection in sexually active young adults. Infection occurs from inoculation of the eye with infected genital secretions. Human herpesvirus 6 (HHV-6) is a common herpesvirus about which little is known. HHV-6 causes a maculopapular rash in infants known as roseola infantum, and may also be a common cause of high fevers in children.

Case A 3-day-old female newborn presents with rapidly progressing bilateral conjunctivitis with white discharge. She was born full term via precipitous vaginal delivery; her mother had no prenatal care. On exam, she is alert and active. The eye discharge is purulent, and she has bilateral eyelid edema. Question Ico-delete Highlights What should be the next step in the management of this newborn?

Correct answer: Admit to the hospital for IV antibiotics and evaluation. Explanation The correct response is to admit the infant to the hospital for IV antibiotics and evaluation. Ophthalmia neonatorum is a form of conjunctivitis occurring in infants younger than 4 weeks. The usual incubation period is 2-5 days for N. gonorrhoeae, and 5-14 days for C. trachomatis. Conjunctivitis due to silver nitrate drops usually occurs 6-12 hours after birth. The condition may also be caused by a virus, most commonly herpes simplex. Due to the timing and severity of conjunctivitis, in conjunction with the fact that the mother had no prenatal care, it is possible that this newborn has gonococcal conjunctivitis. Giving an IM dose of ceftriaxone is not enough. Due to the possibility of blindness, gonococcal meningitis, or sepsis, the patient should be monitored closely and admitted to the hospital for eye, blood, and (possibly) CSF cultures. Tests for other infections (e.g., C. trachomatis, syphilis, hepatitis B, and HIV) should be performed.

Case A 25-year-old male medical resident presents with 'acute red eyes' with copious watery discharge. He also notes some aversion to bright light. While rubbing his left eye, he describes a sensation of a 'gritty' foreign body. Other than the aversion to bright light, he denies any visual disturbance or pain. On physical exam, the conjunctiva of both eyes are injected and mildly edematous. The remainder of the exam is within normal limits. Question What is the most likely diagnosis?

Correct answer: Allergic conjunctivitis Explanation Allergic conjunctivitis is almost always secondary to environmental allergens; therefore, it usually presents with bilateral symptoms. The other hallmark symptom would be pruritus, which might be suggested by the rubbing of his eye. Infections caused by bacteria, including chlamydial organisms, are transmissible by eye-hand contact. Often, these infections initially present in 1 eye, with the 2nd eye becoming involved a few days later. Bacterial infections present with a purulent discharge, as opposed to a clear one. Contact lens abuse would often lead to a purulent discharge; it is often unilateral. Hyperacute conjunctivitis has an abrupt onset, and it is characterized by a copious yellow-green purulent discharge that reaccumulates after being wiped away. The symptoms of hyperacute conjunctivitis, which are typically rapidly progressive, also include redness, irritation, and tenderness to palpation. Patients demonstrate marked conjunctival injection, conjunctival chemosis (excessive edema), lid swelling, and tender preauricular adenopathy.

Case A 72-year-old woman presents with a severe unilateral headache, jaw pain, and scalp tenderness. The patient states the headache is of a piercing quality; her jaw hurts only when she chews, and feels better a few minutes after she stops chewing. A pulsation of the temporal artery on the same side as her headache cannot be appreciated, and prednisone is prescribed until the patient can see a specialist. Question What is the reason prednisone is prescribed in this case?

Correct answer: Because the patient is at risk for blindness Explanation The clinical picture is suggestive of giant cell (temporal) arteritis; this is a systemic arteritis affecting the medium and large blood vessels (arteries) in patients over 50 years of age. It is often referred to as temporal arteritis because it frequently involves the extracranial branches of the carotid artery. Signs and symptoms include headache, scalp tenderness, jaw claudication, visual problems, and throat pain. If not treated appropriately, blindness can occur due to the involvement of the posterior ciliary branch of the ophthalmic artery. Prednisone should be initiated immediately. This patient has a vasculitis, not a stenosis, of the artery on the same side as the headache; contralateral is the opposite side. Pseudotumor cerebri is intracranial hypertension with similar symptoms, but treatment is with diuretics and weight loss. A berry aneurysm is an out-pouching of a cerebral blood vessel wall. A typical symptom is described as "the worst headache of my life" by patients. Some symptoms are similar to a temporal arteritis, but treatment is surgical. Angioedema of the oropharynx is usually due to an allergic reaction to numerous agents. Common symptoms are difficulty breathing and throat fullness as a result of airway occlusion. Treatment is the removal of the offending agent and securing the airway (if indicated).

Case A 74-year-old man presents with a 1 ½-hour history of severe pain and blurred vision in his left eye. Upon examination, his left eye is erythematous with a steamy cornea and a nonreactive, dilated pupil. An ophthalmologic consult is ordered, and tonometry is completed, revealing an elevated intraocular pressure and a confirmed diagnosis of acute angle-closure glaucoma. Question Ico-delete Highlights What will be the definitive treatment for this patient?

Correct answer: Bilateral laser peripheral iridotomy Explanation The correct answer is bilateral laser peripheral iridotomy. This is a procedure during which a puncture-like opening is made near the base of the iris in order to decrease intraocular pressure in patients with angle-closure glaucoma. While there are various medications used to treat acute episodes, this procedure will correct the disorder definitively, whereas the medications are temporary treatment. Patients with narrow anterior chambers are at risk for angle-closure glaucoma. If this occurs unilaterally, they are even more at risk for acute episodes in the other eye. For this reason, the procedure is typically performed bilaterally. Left laser peripheral iridotomy is not the correct answer. While there are various medications used to treat acute episodes of angle-closure glaucoma, this procedure will correct the disorder definitively, whereas the medications are temporary treatment. Patients with narrow anterior chambers are at risk for angle-closure glaucoma. Narrow anterior chambers always occur bilaterally. If acute angle-closure glaucoma occurs unilaterally, they are even more at risk for acute episodes in the other eye. For this reason, the procedure is typically performed bilaterally as opposed to being done in JUST the affected eye. IV acetazolamide is not the correct answer. This medication is given in episodes of acute angle-closure glaucoma in order to decrease the intraocular pressure. It is typically given in a single 500mg IV dose followed by 250mg orally 4 times daily. This is effective to control the acute episode, but will not treat the disorder definitively, as the patient's underlying issue is narrow anterior chambers. Oral glycerol is not the correct answer. This medication is an osmotic diuretic that can be given 1-2 g/kg in order to decrease a patient's intraocular pressure during an acute episode of angle-closure glaucoma. This is effective to control the acute episode, but will not treat the disorder definitively, as the patient's underlying issue is narrow anterior chambers. Topical timolol 0.25% is not the correct answer. This medication is a topical β-adrenergic blocking agent used twice daily chronically in patients who have chronic glaucoma. The disorder does not require the acute lowering of intraocular pressure such as angle-closure glaucoma. Topical timolol would not be effective in lowering intraocular pressure in patients with angle-closure glaucoma.

Case A 52-year-old man presents with a burning sensation of both eyes. He denies recent trauma and no contact with individuals who have similar symptoms. On examination, his eyelid margins are red and inflamed. His eyelashes are greasy and adherent with a surrounding dandruff-like scale. Conjunctiva are clear. Question What is the most likely diagnosis?

Correct answer: Blepharitis Explanation Blepharitis is a chronic inflammation of the lid margins. Causes include infection, seborrhea, and meibomian gland dysfunction. Blepharitis characteristically causes red eyelid margins with adherent eyelashes and scale-like deposits. It is treated with eyelid scrubs with diluted baby shampoo. Antibiotics may be used if infection is suspected. Hordeolum presents as a small inflammatory nodule in a gland on the upper or lower eyelid margin. Chalazion presents as a painless lesion of the palpebral margin. Conjunctivitis presents in several different ways depending on the etiology. However, the conjunctiva would be erythematous; this patient's conjunctiva are clear Pinguecula presents as an elevated yellow fleshy mass on the sclera adjacent to the conjunctiva.

Question What description of "red eye" is caused by acute glaucoma?

Correct answer: Dilated central and conjunctival vessels, dilated pupil, and decreased vision Explanation Acute glaucoma is the sudden increase in intraocular pressure secondary to blocked drainage from the anterior chamber. It manifests as dilatation of both the central and conjunctival vessels. Pupil size is often dilated, and vision is decreased. It is considered an ocular emergency. Conjunctival injection is the dilatation of the conjunctival vessels. Pupil size and vision are unaffected. It is a result of superficial processes, such as infection, allergies, irritation, and vasodilators. Ciliary injection is the dilatation of the branches of the anterior ciliary artery, and it manifests as a diffuse flush and dilatation of the central vessels around the iris. The pupil may be normal, but it is usually small and anisocoric. Vision is decreased. It is a result of disorders of the cornea or inner eye, and requires immediate attention. A subconjunctival hemorrhage is a patch of blood that appears outside the vessels between he conjunctiva and sclera. Pupil size and vision are unaffected. It may be the result of trauma, bleeding disorders, or sudden increases in venous pressure, such as from coughing. It is usually not of clinical significance. Blepharitis is inflammation of the eyelid margins. Vessels, pupil size, and vision are all normal. Seborrhea and staphylococcal infections are common causes.

Case A client presents with a 1-year history of flaking and scaling around her lashes, along with itching and a burning sensation. She has also noted her lid margins are red, and some of her lashes are missing. Her history is significant for seborrheic dermatitis of the scalp, eyebrows, and external ears, and diabetes. Question What is the most likely diagnosis?

Correct answer: Blepharitis Explanation Blepharitis may be seborrheic or ulcerative. Seborrheic (non-ulcerative) blepharitis is commonly associated with seborrhea of the face, eyebrows, external ears, and scalp. Inflammation of the eyelid margins occurs, with redness, thickening, and often the formation of scales and crusts or shallow marginal ulcers. Ulcerative blepharitis is caused by bacterial infection (usually staphylococcal) of the lash follicles and the meibomian glands. Removal of crusts, topical antibiotics, and /or oral antibiotics remains the mainstay of treatment. Chalazion is a mildly painful swelling of the eyelid margin due to granulomatous inflammation, and usually resolves spontaneously with warm soaks and time. Multiple chalazion are seen in diabetes, alcoholism, and malnourishment. In extreme cases curettage may be required. Conjunctivitis can be caused by bacterial, viral, allergic, and irritant etiologies. Clients usually complain of red eyes and a sticky or watery discharge. The type of discharge helps to determine the etiology; it is watery in viral, and sticky green or yellow in bacterial conjunctivitis. Irritation is common, but severe pain and photophobia are not. Bacterial or viral conjunctivitis is usually self-limited, but it may be treated with a topical antibiotic, without steroids, such as sulfacetamide (10% 3-4 times/day). Topical aminoglycoside should be reserved for more refractory disease. Allergic conjunctivitis may be effectively treated with a new class of non-steroidal, topical, anti-inflammatory agents. Irritant conjunctivitis, including dry eyes, may be treated with topical, non-preserved lubricants. Ectropion is the outward turning of, usually, the lower lid occurring in older people. Surgery is indicated if ectropion causes excessive tearing, exposure, keratitis, or a cosmetic problem. Foreign body sensation is most commonly due to corneal or conjunctival foreign bodies. Other causes are disturbances of the corneal epithelium and rubbing of eyelashes against the cornea (trichiasis).

Question A 3-year-old boy presents after waking up with his eyelids glued together. His mother states that he has been rubbing his eyes constantly. The medical history is unremarkable, and the patient had not been sick before. Upon examination, redness of the lid margin, edema, conjunctival irritation, and loss of lashes can be seen. There are also some scales on the lid margin that can be removed easily. What is the most likely diagnosis?

Correct answer: Blepharitis Explanation The symptoms described above are typical for blepharitis, an inflammation of the lid margins. There are 2 types of blepharitis: Ulcerative blepharitis: it is caused by bacterial infection (mostly staphylococci) of follicles and meibomian glands. Adherent crusts develop that result in bleeding if removed. Pustules form in the lash follicles and turn into small ulcers. Repeated episodes can lead to permanent loss of lashes, scarring of the lids, and sometimes even corneal ulceration. Treatment consists of application of antibiotic ointment for 7 to 10 days. Seborrheic blepharitis: the cause is not known. Sometimes it is associated with seborrhea of scalp and face. The scales are easily removable. Treatment is usually to just keep the eyelids clean. If it does not clear, antibiotic ointment is needed. Hordeolum is a localized infection of the Zeis, Moll, or meibomian glands; it is usually caused by staphylococci. It can be associated with blepharitis or follow it. The internal hordeolum (affecting the meibomian glands) involve pain, redness, and localized edema with an elevated yellow area near the affected gland that turns into an abscess, which rarely ruptures spontaneously. The external hordeolum (affecting the glands of Zeis or Moll) starts with pain, redness, and tenderness, which turns into an induration with a yellowish spot in the center. The abscess soon ruptures and emits pus. Acute conjunctivitis is an inflammation of the conjunctiva that can be bacterial, viral, or allergic. The most common pathogens for bacterial infection in adults are Staphylococci, Streptococcus pneumoniae, and Haemophilus influenzae. In children, bacterial conjunctivitis is more common than viral conjunctivitis and is caused by Streptococcus pneumoniae, Haemophilus influenzae, and Moraxella catarrhalis. Symptoms are itching, irritation, foreign body sensation, tearing, and in bacterial infection, mucopurulent discharge. Small bumps with fibrovascular cores on the palpebral conjunctiva that look like a velvety surface are a typical papillary reaction. Bacterial conjunctivitis is usually self-limiting but can also cause corneal or systemic complications (like meningitis in primary meningococcal meningitis) and should therefore be treated with topical antibiotics (e.g., polymyxin-bacitracin, ciprofloxacin, or ofloxacin). Viral conjunctivitis is more common than bacterial conjunctivitis in adults; it is caused by herpes virus or during the course of systemic or cutaneous infections, such as rubella, measles, cytomegaly, syphilis, or systemic adenovirus infections. Occasionally permanent vision loss can occur, as well as cataract, microphthalmos, retinal involvement, interstitial keratitis, and optic neuritis. Trachoma is a chronic conjunctivitis caused by chlamydia trachomatis. It is transmitted via contact (hands, towels, etc.), and it is a major cause of blindness in developing countries due to poor hygiene and economic conditions. In the United States, only mild forms are typically seen, usually in immigrants from endemic areas. The disease presents as conjunctivitis with small lymphoid follicles. Cultures are often false negative; therefore, PCR to prove the presence of chlamydial DNA in ocular secretions should be performed. Treatment consists of topical or systemic tetracycline or erythromycin.

Case A 66-year-old man presents with sudden onset of brief episodes of blindness in his right eye, with complete recovery of vision within 24 hours. The event is described as a shade coming down across his field of vision, and it is not painful. Question What is the most common underlying condition leading to the event described above?

Correct answer: Carotid stenosis Explanation The diagnosis for this case is amaurosis fugax. The most common cause of amaurosis fugax is ipsilateral carotid stenosis, leading to carotid emboli that lodge in the retinal arteries, causing temporary blindness until the emboli dissolves. Screening is performed through carotid Doppler ultrasonography, CT, or MR angiography. Treatment includes low-dose aspirin or other anti-platelet therapy, along with treatment of underlying cause. Migraine headaches, sickle cell disease, and acute angle-closure glaucoma are all less common causes of amaurosis fugax. Lower extremity deep venous thrombosis is not a likely source for emboli causing amaurosis fugax.

Case An 84-year-old man presents to the emergency room with 2.5-hour history of painless, progressive vision loss in his right eye, which began while he was reading. He denies any other symptoms. Past medical history is positive for hypertension and a cardiac dysrhythmia. Physical exam findings include a BP of 180/110 mm Hg; other vital signs are normal. The right eye non-reactive to light. Funduscopic exam reveals a pale retina with a red spot. The rest of the physical exam is normal. There is a "box-car" like segmentation of blood in the retinal veins. Question What is the most likely cause of vision loss in this patient?

Correct answer: Central retinal artery occlusion Explanation Central retinal artery occlusion (CRAO) interferes with the major supply of blood to the retina, causing vision loss. In 25% of individuals, the macula is supplied by the cilioretinal arteries, sparing some central vision in the event of CRAO. The occlusion is principally caused by thrombus, thromboemboli, cholesterol plaques, calcium, or vasospasm. The patient notes a sudden, painless, monocular vision loss; the physical examination reveals a problem in the visual afferent way, and the funduscopic examination shows the red spot, which is the pigment of the choroid showing through the macula. There is also a characteristic "box-car" like segmentation of blood in the veins. Treatment consists of decreasing intraocular pressure in order to increase the pressure gradient in the artery and force the embolus to dislodge, restoring vision. Application of digital pressure, carbonic anhydrase inhibitors, beta-blockers, and paracentesis of the anterior chamber are some of the methods used for this purpose. Retinal vein occlusion should be considered in the differential; it is characterized by preservation of some vision, and it rarely shows a red spot in the retina at the funduscopic examination. Retinal detachments often cause prodromal symptoms, such as flashing lights, floating 'spider webs', and the sensation of 'having a curtain drawn up or down' over the visual field. The funduscopic exam reveals an undulating, pale, detached retina. Acute angle-closure glaucoma is characterized by severe ocular pain and blurred vision, rather than loss of vision.

Case Ico-delete Highlights A 78-year-old Caucasian man presents with unilateral painless loss of vision in the right eye of 3 hours duration. Examination reveals an elderly man who is anxious but in no acute distress. Visual acuity is light perception only in the right eye and 20/30 in the left eye. Pupillary examination is significant for an afferent pupillary defect on the right side. Penlight examination of the eyes is otherwise unremarkable. Retinal examination of the right eye reveals a cherry-red spot. Retinal examination of the left eye is unremarkable. Question Ico-delete Highlights What disease process most likely accounts for the patient's presentation?

Correct answer: Central retinal artery occlusion Explanation This case represents the classic presentation of a central retinal artery occlusion. Namely, it is acute, unilateral, and painless; there is loss of vision as well as a cherry-red spot on fundus examination. Tay-Sachs disease is a lysosomal storage disease found predominantly in Ashkenazi Jews. Infants with this fatal neurodegenerative disease do have a cherry red spot on retinal examination. An adult-onset form of Tay-Sach's disease is rare but does exist. Onset of symptoms is in the 3rd or 4th decade of life; however, it is characterized by neurologic deterioration and cherry-red spots would be bilateral. Although this patient does have a cherry-red spot, there is nothing in the presentation to suggest Tay-Sachs disease. Open angle glaucoma is a chronic, slowly progressive condition that would not be expected to cause acute visual loss. The patient gives no history of trauma, and no evidence of trauma is seen on examination. A cataract, or opacification of the lens, would not be expected to cause acute visual loss.

Case A 24-year-old man presents with a painless, localized swelling of his left lower eyelid; it has developed over a period of weeks. He is seeking medical attention because it is now producing a foreign body sensation in his left eye; it is also hindering his path of vision. On physical examination, his visual acuity is normal; there is no evidence of injection or discharge. You palpate, and you observe a nontender, localized nodule on the lower eyelid. Question What is the most likely diagnosis?

Correct answer: Chalazion Explanation Chalazion is the correct response. Chalazion is a granulomatous inflammation of the meibomian gland, and it has the typical presentation of being a hard, nontender swelling of either the upper or lower lid. It is sometimes accompanied by redness and swelling of the adjacent conjunctiva. Blepharitis is the incorrect choice; typically, blepharitis is a bilateral condition of general inflammation of the eyelid skin, eyelashes, and associated glands. Characteristics of blepharitis include red-rimmed eyes as well as scales on the eyelashes. Tears may even have a greasy distinction to them. Ectropion is also an incorrect choice. Ectropion is the outward turning of the lower lid. This is not what was described in this case. Keratitis is also incorrect; it produces a painful eye, a hazy-appearing cornea as well as evidence of an ulcer or even an abscess in this area; hypopyon is also possible in patients who have keratitis. Malignant melanoma can have a presentation in the ocular area that is somewhat like the one described in this case; however, it is not a common presentation.

Question A client comes into the clinic with a complaint of a hard, non-tender swelling on the upper lid of her left eye. The conjunctiva in the region of the lesion is red and elevated. What is the most likely diagnosis?

Correct answer: Chalazion Explanation Chalazion is a mildly painful swelling of the eyelid margin, due to granulomatous inflammation, and usually resolves spontaneously with warm soaks and time. Multiple chalazion are seen in diabetes, alcoholism, and malnourishment. In extreme cases curettage may be required. Blepharitis may be seborrheic or ulcerative. Seborrheic (non-ulcerative) blepharitis is commonly associated with seborrhea of the face, eyebrows, external ears, and scalp. Inflammation of the eyelid margins occurs, with redness, thickening, and often the formation of scales and crusts, or shallow marginal ulcers. Ulcerative blepharitis is caused by bacterial infection (usually staphylococcal) of the lash follicles, and the meibomian glands. Conjunctivitis can be caused by bacterial, viral, allergic, and irritant etiologies. The clients usually complain of red eyes, and a sticky or watery discharge. The type of discharge determines the etiology-watery in viral, and sticky green or yellow in bacterial conjunctivitis. Irritation is common, but severe pain and photophobia are not. Bacterial or viral conjunctivitis is usually self-limiting, but it may be treated with a topical antibiotic, without steroids, such as sulfacetamide (10% 3-4 times/day). Topical aminoglycoside should be reserved for more refractory disease. Allergic conjunctivitis may be effectively treated with a new class of non-steroidal, topical, anti-inflammatory agents. Irritant conjunctivitis, including dry eyes, may be treated with topical, non-preserved lubricants. Ectropion is the outward turning of, usually, the lower lid occurring in older people. Surgery is indicated if ectropion causes excessive tearing, exposure, keratitis, or a cosmetic problem. Foreign Body sensation is most commonly due to corneal or conjunctival foreign bodies. Other causes are disturbances of the corneal epithelium, and rubbing of eyelashes against the cornea (trichiasis)

Case A 23-year-old woman presents with a 2-week history of left eye irritation. She insists that there is a foreign object lodged under her eyelid. There are no other symptoms. Physical examination is remarkable for unilateral conjunctivitis, which appears acute and follicular in presentation, and mucoid discharge. There is no detectable foreign object present under the eyelid. 4 weeks ago, the patient presented with abdominal discomfort and vaginal bleeding. A cervical culture was obtained, and a blood specimen was obtained for a CBC. No other remarkable findings were noted on her last visit. Cervical cultures came back negative. CBC and differential were within normal limits. She had previously denied any sexual activity. When questioned again, she reveals that she has been sexually active with 2 male sexual partners in the past 4 months. The laboratory on a conjunctival specimen performs a DFA test, and the result is as follows (see image). A cervical specimen is also collected for DFA, with similar results. All routine cultures for both sites are negative for any significant organisms or pathogens, and Gram stain results are "NOS". Question The most likely diagnosis is inclusion conjunctivitis concurrent with cervicitis due to what disease?

Correct answer: Chlamydia trachomatis Explanation Chlamydia trachomatis is an obligate intracellular parasite with a unique biphasic life cycle. It does not Gram stain; laboratory procedures used for diagnosis include isolation in tissue culture, EIA detection of antigen, immunofluorescent staining, cytologic examination for intracytoplasmic inclusions, and by the demonstration of nucleic acid by direct hybridization or by amplification techniques. It can cause inclusion conjunctivitis and ocular trachoma (as well as urethritis, lymphogranuloma venereum, urogenital infections, infertility, salpingitis and endometritis, reactive arthritis, etc.). The inclusion conjunctivitis presents as an acute follicular conjunctivitis and is usually self-inoculated from an infected genitourinary site. The patient frequently notes a foreign body presence in the eye. These symptoms are usually unilateral, and in the first 2 weeks, there is a mucoid discharge that becomes purulent. Usually the inclusion conjunctivitis resolves without complications, but some untreated or improperly treated cases can result in a prolonged infection that can last for months, and it can produce conjunctival and corneal scarring that is similar to mild ocular trachoma. Antibiotics, such as the tetracyclines, macrolides, rifampin, and some of the fluoroquinolones, have activity against chlamydia. Pseudomonas aeruginosa is a Gram-negative rod; it is a non-lactose fermenting, oxidase-positive motile bacteria. Growth on MacConkey agar is usually characterized by the production of a "grape-like" smell. A blue-green color, due to the production of the diffusible fluorescent pigments pyoverdin and pyocyanin, is characteristic of the colonies growing on MacConkey. Pseudomonas aeruginosa is a very common opportunistic source of human infections, especially in the hospital setting. Pathogenesis is due to its minimal nutritional requirements, relative resistance to antibiotics, and a host of other invasive and toxicogenic substances that it produces. It can cause a keratitis that is rapid in its development. The infection is usually the result of a previous injury to the eye, which causes an interruption in the epithelial surface and allows bacterial invasion of the underlying stroma. It can also be caused by contact lenses. Fever is usually absent, and leukocytosis is absent or minimal. The infection can lead to corneal ulceration, resulting in the rapid loss of ocular function; therefore, these infections need to be approached as a medical emergency. Scrapings from the floor of the ulcer exhibiting Gram-negative rods are strongly indicative of Pseudomonas aeruginosa and should necessitate treatment. Immediate initiation of combined topical and subconjunctival therapy with an aminoglycoside antibiotic such as gentamicin or tobramycin is advised. Topical steroids are sometimes used to reduce ocular inflammation. Haemophilus aegyptius is a Gram-negative coccobacillus; it is non-motile, fastidious bacteria requiring the presence of special factors for its growth on agar media. These factors are hemin and nicotinamide adenine dinucleotide, which are present in chocolate agar. The organism is indigenous to humans. It is an important cause of a purulent conjunctivitis called pink eye, and it can occur in outbreaks because of its contagious nature. The diffuse pink color of the sclera and the presence of a serous or purulent discharge are virtually diagnostic of Haemophilus aegyptius infection. Leukocytosis is absent. The infection is not acute in presentation. The treatment of Haemophilus aegyptius is with topical antibiotics. Because of the infectious nature of the infection, instructions should be provided to the patient to help prevent the spread of the infection to others. Bacillus cereus is a Gram-positive (or Gram-variable) rod that is aerobic and spore forming; it is ubiquitous in nature. Bacillus cereus is an important cause of food poisoning. It has also been recognized as an ocular pathogen. The ocular infection is acute in presentation and requires aggressive intervention to save the eye. It is many times associated with metal-on-metal projectile injuries, soil and dust contamination as seen in rural farm areas, and drug abuse. The presence of progressive corneal deterioration and ring abscess formation is a complication of panophthalmitis caused by Bacillus cereus. Except for infections with Pseudomonas aeruginosa, this finding is almost pathognomonic of Bacillus cereus. Because of the seriousness of the infection, early diagnosis is important. Patients presenting with ocular infections after trauma or in the setting of drug abuse should arouse suspicion. As with Pseudomonas aeruginosa, it is important for the prompt initiation of therapy before permanent structural changes occur, leading to loss of vision. Clindamycin and gentamicin in combination, administered intravitreally, is the course of therapy favored by ophthalmologists. The prognosis is poor and usually results in the loss of the eye unless an aggressive approach is undertaken; even then, there is assuredly some loss of vision. Topical steroids are sometimes used to reduce ocular inflammation. Acanthamoeba is a free-living amebae that can cause granulomatous amebic encephalitis and keratitis. Detection is usually made by observing the free-living motile organisms in a wet prep preparation. Acanthamoeba keratitis is a corneal infection that occurs in healthy people and is usually associated with contact lens wearers. To prevent Acanthamoeba keratitis, it is recommended that contact lenses be cleaned and stored with Benzalkonium chloride-preserved saline and solutions containing thimerosal with edetate. Swimming in fresh water (where the organism is naturally found) with contact lenses can predispose the wearer to Acanthamoeba keratitis. The keratitis is slow in developing and is frequently mistaken for herpes, bacterial, or fungal keratitis. Frequently, the average delay to definitive treatment can range from days to months. Symptoms include blurred vision, conjunctivitis, tearing, severe pain to the eye, and photophobia. The keratitis achieves an advanced stage in several days to several months, and it can exhibit patchy stromal infiltrates and dendriform epithelial involvement without frank corneal ulceration in its early stages. A ring corneal infiltrate is characteristic of this keratitis in its late stages. Early diagnosis, aggressive surgical debridement, and medical management can prevent eye damage. High concentrations of topical antimicrobial drugs (1% miconazole, 0.1% propamidine isethionate, and Neosporin) for a minimum of 3 - 4 weeks is part of the antibiotic therapeutic regimen employed in the treatment of Acanthamoeba keratitis. Staphylococcus aureus is a Gram-positive staining cocci that is catalase positive and coagulase positive. It is probably the 2nd most common bacterial isolate of human infections behind Escherichia coli and the most common cause of bacterial endophthalmitis. Staphylococcus aureus has a host of invasive and toxigenic characteristics that enhance the pathogenesis of the organism in the human host. The organism has been described as an etiologic agent of many infections including, but not limited to, conjunctivitis, endocarditis, septicemia, abscesses, and urinary tract infections. The conjunctivitis caused by Staphylococcus aureus is usually characterized as non-severe where there is little to no lid edema, scant purulent discharge, and normal cornea; however, in some cases the presentation can be severe. Topical agents are usually used to treat this infection such as cephalosporins or semisynthetic penicillins. In suspected cases of resistance, topical vancomycin should be considered. Candida albicans is a yeast. Yeast appear on Gram stain as large Gram-positive organisms that are approximately 3 - 5 times larger than Gram-positive cocci. They are aerobic and generally grow well on most non-selective agar media. Endophthalmitis due to yeast is generally a common and serious complication of intravenous drug use. Candida albicans is the most common fungal cause. It is usually of hematogenous origin, where the patient has infective endocarditis or some other infective process occurring. The symptoms are blurred vision, decreased vision, white cotton appearing exudative lesions in the choroid and retina with vitreous haziness, and eye pain. A definitive diagnosis is made by obtaining vitreous fluid for Gram stain and culture. The treatment consists of parenteral amphotericin B together with flucytosine. Intraocular amphotericin B administration as therapy is controversial. The incidence of permanent intraocular damage is high.

Case A 73-year-old African-American man presents for a routine follow-up since being diagnosed with glaucoma. His ophthalmologist has started him on a topical carbonic anhydrase inhibitor (dorzolamide). Question What is the site of action of this drug?

Correct answer: Ciliary body epithelium Explanation The correct response is the ciliary body epithelium. Carbonic anhydrase reduces intraocular pressure by reducing the synthesis of aqueous humor. Aqueous humor is produced in the epithelium of the ciliary body; it travels through the pupil into the anterior chamber and drains through the trabecular meshwork into the canal of Schlemm. A topical drug that decreases aqueous humor production would be expected to act at the ciliary body epithelium. The trabecular meshwork and canal of Schlemm are part of the outflow pathway of aqueous humor; they are not involved in aqueous humor production. The corneal endothelium and retinal pigment epithelium are not involved in aqueous humor production.

Case A 22-year-old woman presents with a 1-day history of foreign body sensation in her right eye. She woke up with pain in the right eye, and she immediately had trouble opening her eye. She wears soft contact lenses and does not remember how long the last pair was in for. She removed her contact lenses the night before the pain started. There was no trauma. Visual acuity was 20/40 O.U. without corrective lenses, and extraocular movements were within normal limits. With fluorescein stain, a defect is noted; it is round and found at the center of the cornea. No foreign bodies are noted. Question What intervention is indicated?

Correct answer: Ciprofloxacin 0.3% solution Explanation The correct answer is ciprofloxacin 0.3%; the patient has a corneal abrasion due to contact lenses. Ciprofloxacin covers pseudomonas, and pseudomonas should always be covered when someone gets a corneal abrasion from contact lenses. A pressure patch is never used in someone who gets a corneal abrasion from contact lenses because of the risk of developing infectious keratitis. Trifluridine drops are used as an antiviral for Herpes Simplex keratitis; they are not used for corneal abrasions. Ketorolac solution is used to help pain in a corneal abrasion, but it is not the mainstay of treatment. Sulfacetamide solution is not used for corneal abrasions caused by contact lenses because it does not cover pseudomonas adequately.

Question A white female comes into the evening clinic with complaint of red eyes with a sticky discharge. Her eye feels irritated, the pain is mild. She tells you she shared towels with her boyfriend who had a red eye. She asks you if you think her boyfriend gave her an infection. What is your working diagnosis?

Correct answer: Conjunctivitis Explanation Conjunctivitis can be caused by bacterial, viral, allergic, and irritant etiologies. Clients usually complain of red eyes and a sticky or watery discharge. The type of discharge helps to determine the etiology-watery in viral, and sticky green or yellow, in bacterial conjunctivitis. Irritation is common, but severe pain and photophobia are not. Bacterial or viral conjunctivitis is usually self-limited, but it may be treated with a topical antibiotic, without steroids, such as sulfacetamide (10% 3-4 times/day). Topical aminoglycoside should be reserved for more refractory disease. Allergic conjunctivitis may be effectively treated with a new class of non-steroidal topical anti-inflammatory agents. Irritant conjunctivitis, including dry eyes, may be treated with topical, non-preserved lubricants. Blepharitis may be seborrheic or ulcerative. Seborrheic (non-ulcerative) blepharitis is commonly associated with seborrhea of the face, eyebrows, external ears, and scalp. Inflammation of the eyelid margins occurs, with redness, thickening, and often the formation of scales and crusts, or shallow marginal ulcers. Ulcerative blepharitis is caused by bacterial infection (usually staphylococcal) of the lash follicles and the meibomian glands. Chalazion is a mildly painful swelling of the eyelid margin, due to granulomatous inflammation, and usually resolves spontaneously with warm soaks and time. Multiple chalazion may be seen in diabetes, alcoholism, and malnourishment. Rarely, curettage may be required. Ectropion is the outward turning of, usually, the lower lid occurring in older people. Surgery is indicated if ectropion causes excessive tearing, exposure, keratitis, or a cosmetic problem. Foreign Body sensation is most commonly due to corneal or conjunctival foreign bodies. Other causes are disturbances of the corneal epithelium and rubbing of eyelashes against the cornea (trichiasis).

Case A 32-year-old woman presents with a 3-day history of irritation, burning, itching, and redness of both eyelids. She denies fever, visual changes, and photophobia. On physical examination, you note the presence of scales clinging to the eyelids bilaterally. Question What is the proper management in this case?

Correct answer: Daily cleaning with a damp cotton applicator and baby shampoo Explanation The scenario presented above depicts a patient with anterior blepharitis, which is a common disorder seen in primary care; it typically consists of a recurrent bilateral inflammation of the lid margins that involves the eyelid skin, eyelashes, and associated glands. Commonly, the underlying cause is seborrhea, which usually originates in the scalp, eyebrows, or ears. Sometimes, anterior blepharitis can be ulcerative, and the origin in the presented case is staphylococci. Anterior blepharitis can typically be resolved and controlled by cleaning the affected areas daily using a damp cotton applicator, warm water, and a baby shampoo mixture. The object of the daily cleaning is to remove the visible scales as efficiently as possible. None of the other listed options are an appropriate treatment plan for anterior blepharitis. Patients can also be diagnosed with what is known as posterior blepharitis, which is an inflammation of the meibomian glands of the eyes. It is usually staphylococcal in origin, and it typically presents with significantly worse signs and symptoms, such as hyperemic lids, the presence of telangiectasias, inflammation of the gland or their orifices, or even abnormal secretions; tears may be described as being frothy or greasy. More significant cases of posterior blepharitis can lead to conjunctivitis, hordeola, chalazions, eyelash trichiasis, or even corneal vascularization and thinning. Treatments for posterior blepharitis may consist of long-term oral antibiotic therapy, short-term topical steroids, or short-term topical antibiotics eye drops; if significant complications are evident, an ophthalmologist referral is indicated.

Case Ico-delete Highlights A 27-year-old woman presents with a 3-day history of left eye pain. The patient notes sensitivity to light, and she comments that her eye throbs in pain at night. On physical examination, there is redness and a loss of visual acuity. There is no drainage from the eye or visible lesions. Slit-lamp exam demonstrates inflammatory cells in the aqueous humor. Question What would be an appropriate treatment for this patient?

Correct answer: Dexamethasone and homatropine ophthalmic drops Explanation The correct response is dexamethasone and homatropine ophthalmic drops. The clinical picture is suggestive of uveitis. Patients with uveitis usually note redness, pain, photophobia, and visual loss. Treatment of anterior uveitis is with topical steroids; posterior uveitis may require oral corticosteroids. There are multiple causes of uveitis, but it is primarily immunogenic and seen in the context of another systemic condition. Cool compresses and artificial tears are not an effective treatment for uveitis. Cortisporin is effective against bacterial conjunctivitis. Patients typically present with a copious discharge in the affected eye with mild discomfort. There is no loss of visual acuity. Oral acyclovir is used in the treatment of herpes simplex keratitis. Dendritic ulcer is seen on staining with fluorescein, which is not seen in this patient. IV acetazolamide is used in the treatment of acute angle-closure glaucoma. Patients typically present with rapid onset of severe eye pain, profound visual loss, and halos around lights. The symptoms of severe eye pain, profound vision loss, and halos around lights are not seen in this patient.

Case A 52-year-old man presents with gradual bilateral deteriorating vision; he was treated at a visit 6 months prior, and was instructed by his ophthalmologist to follow-up in 6 months. The patient has a 15-year history of NIDDM and is on an oral hypoglycemic (sulfonylurea drug); his blood sugar level is usually well-maintained. He has some skin trophic changes in the lower limbs without impairment of sensation. Vision is 6/36 bilaterally. The lens is clear, and the vitreous shows no abnormalities. The image shown is the fundus of the patient's right eye. Question What is the most likely diagnosis?

Correct answer: Diabetic retinopathy Explanation Diabetic retinopathy is 1 of the most common complications of diabetes. Diabetes is well-known to cause microvasculopathy and neuropathy. Vasculopathy affects the kidneys and the retina of the eye. Retinopathy is characterized by neovascularization and retinal hemorrhages, and may also lead to fibrosis, adhesions, and subsequent retinal detachment. Early detection, prevention, and treatment help to retain vision. The most widely used treatment is panretinal photocoagulation (PRP) with a laser; it is always applied to the periphery of the retina, outside the arcades. As shown in the image, the white dots are the scars of the laser; the center shows the minute retinal hemorrhages and the neovascularization. Diabetic macular edema is characterized by marked, rapid, defective vision, usually in 1 eye, which may be followed by the other eye. The image shows central yellow dots, or hemorrhages, and is treated with central laser or intravitreal injection with steroids. Central retinal vein occlusion may occur in diabetics; it is usually unilateral, and the retina shows congested veins and marked hemorrhages. It may be treated with PRP. Central retinal artery occlusion causes unilateral sudden blindness; it may be due to an embolus or thrombus, and the fundic picture shows a pale white retina with a central cherry red spot. Age-related macular degeneration occurs in older people, usually after age 70. It may be exudative with central edema and exudates, or dry with macular scarring. It causes gradual diminution of vision.

Case A 35-year-old woman presents with a 24-hour history of purulent drainage and erythema of her right eye. After a brief physical examination, cultures of the drainage are taken and she is started on a medication prophylactically that would cover the most common bacterial causes of conjunctivitis (including sexually transmitted diseases). Question On what medication (in either an oral or topical form) would she most likely be started?

Correct answer: Erythromycin Explanation Erythromycin ophthalmic ointment applied 2-4 times daily is a treatment option for non-sexually transmitted bacterial conjunctivitis. If trying to cover all bacterial etiologies of conjunctivitis, then erythromycin can be given in the oral form in order to include good coverage for both gonococcal conjunctivitis and chlamydial conjunctivitis. If the erythromycin ophthalmic ointment were to be prescribed in a patient with a sexually transmitted bacterial conjunctivitis, there may still be a partial or complete resolution of symptoms. Erythromycin, in either the topical or oral form, has a good chance of treating any bacterial cause of conjunctivitis until the culture results confirm the etiologic agent. Tetracycline 250 mg po 4 times daily for 3 weeks is a good treatment choice for chlamydial conjunctivitis if the cultures reveal this as the cause, but it would not be a good prophylactic choice while waiting for lab results. Bacitracin ophthalmic ointment applied 2-4 times daily for 5 days is a good treatment option for patients with bacterial conjunctivitis that is not from a sexually transmitted disease. In these cases, the most common etiologic agent is Staphylococcus aureus. Olopatadine is an antihistamine ophthalmic solution that is used in the treatment of ocular itching associated with allergic conjunctivitis. It would not be of any help in a patient with bacterial conjunctivitis, regardless of the etiology. Acyclovir is an antiviral that is prescribed 400 mg po 5 times a day for 7 days in cases of herpetic viral conjunctivitis. It would not be of any help in a patient with bacterial conjunctivitis, regardless of the etiology.

Case A 25-year-old man presents with a 1-day history of redness, itching, and swelling in the right upper eyelid. The redness has increased; there has been no eye discharge or watering. On examination, a pustular swelling at the root of the lashes is found. It is surrounded by redness, and it is tender on palpation. The lesion is shown in the image. Question What is the most likely diagnosis?

Correct answer: External hordeolum Explanation There are 2 types of hordeolum. External hordeolum is an acutely inflamed stye caused by an infection of the glands of Zeis (sebaceous glands near the eyelashes) or glands of Moll (apocrine glands of the lid margin). This causes redness, irritation, and itching, followed by a white dot surrounded by erythema. Warm compresses, plus erythromycin cream, is usually effective in most cases. The other type is internal hordeolum, which is a secondary infection of the meibomian glands in the tarsal plate. This is caused by obstruction of the gland orifice, and causes a lid swelling that typically diminishes in size with voluntary squeezing of the eyelid. It may be acutely inflamed and cause diffuse lid swelling and erythema. Commonly, there is remission and exacerbation. The treatment of the acute form is systemic antibiotics, hot fomentation, and topical erythromycin cream; the chronic persistent form usually requires surgical removal and curettage from the conjunctival side. Chalazion is a granuloma of the meibomian gland, and it is usually painless. It may occur spontaneously, or may develop from an internal hordeolum. Blepharitis is inflammation of the lid margin, either due to allergy or infection. It is mostly associated with dry secretions, scales, and redness; there may be itching. Infective cases are treated with antibiotic drops; allergic blepharitis is usually treated with local steroids. Conjunctivitis is inflammation of the conjunctiva and is always associated with red eye and copious eye discharge. Topical antibiotics are used to treat infective conjunctivitis, while steroids and mast cell stabilizers are used to treat the allergic type. Herpes zoster ophthalmicus is an infection due to herpes zoster; it affects the ophthalmic division of the trigeminal nerve. It causes a rash on the upper eyelid that is extremely painful, unilateral, and may affect the cornea causing keratitis which presents with photophobia, lacrimation, and severe pain. It is typically treated with topical antivirals and steroids.

Case A 25-year-old man is on a skiing vacation. After a few hours of sleep the third night, he awakens with severe bilateral eye pain associated with lacrimation and photophobia. Question What procedure or ocular maneuver would give you the most information to assist in your diagnosis?

Correct answer: Fluorescein staining Explanation The clinical picture is suggestive of ultraviolet keratitis (snow blindness). Slit lamp examination after instillation of fluorescein will show damage to surface cells on the cornea. Visual acuity should be done prior to any testing, but it does nothing to confirm the diagnosis. Testing extraocular movements (EOM) will not confirm the diagnosis. Everting the eyelids is generally done to look at the tarsal plates. Acute angle-closure glaucoma, tested with the Schiotz tonometer, would cause unilateral globe pain and a firm globe.

Question A white male comes into the ER with the complaint of pain in his right eye. He was whittling and felt like something went into his eye. This happened four hours ago. He has washed his eye with water, but it still feels funny. What is your working diagnosis?

Correct answer: Foreign Body Explanation Foreign Body sensation is most commonly due to corneal or conjunctival foreign bodies. Other causes are disturbances of the corneal epithelium and rubbing of eyelashes against the cornea (trichiasis). Blepharitis may be seborrheic or ulcerative. Seborrheic (non-ulcerative) blepharitis is commonly associated with seborrhea of the face, eyebrows, external ears, and scalp. Inflammation of the eyelid margins occurs, with redness, thickening, and often the formation of scales and crusts, or shallow marginal ulcers. Ulcerative blepharitis is caused by bacterial infection (usually staphylococcal) of the lash follicles and the meibomian glands. Chalazion is a mildly painful swelling of the eyelid margin, due to granulomatous inflammation, and usually resolves spontaneously with warm soaks and time. Multiple chalazion may be seen in diabetes, alcoholism, and malnourishment. Curettage may be required in extreme cases. Conjunctivitis can be caused by bacterial, viral, allergic, and irritant etiologies. Clients usually complain of red eyes, and a sticky or watery discharge. The type of discharge helps to determine the etiology-watery in viral, and sticky, green, or yellow, in bacterial conjunctivitis. Irritation is common, but severe pain and photophobia are not. Bacterial or viral conjunctivitis is usually self-limited, but it may be treated with a topical antibiotic, without steroids, such as sulfacetamide (10% 3-4 times/day). Topical aminoglycoside, gentamycin drops, should be reserved for more refractory disease. Allergic conjunctivitis may be effectively treated with a new class of non-steroidal topical, anti-inflammatory agents. Irritant conjunctivitis, including dry eyes, may be treated with topical, non-preserved lubricants. Ectropion is the outward turning of, usually, the lower lid occurring in older people. Surgery is indicated if ectropion causes excessive tearing, exposure, keratitis, or a cosmetic problem.

Case A 62-year-old woman with a history of breast cancer and rheumatoid arthritis presents with stiff neck and severe headache that started a couple of days ago; symptoms are worsening. Upon awakening the morning of presentation, she felt nauseated and vomited twice. Your neurological examination shows right-sided hemiparesis and dilated and nonreactive left pupil. Question What will be your next step?

Correct answer: Fundoscopic examination Explanation Having a history of breast cancer and signs of possible intracranial hypertension, your patient may have metastatic tumor in the brain. Right-sided hemiparesis and dilated nonreactive pupil on the left point to the compression from the left side. While in the office, she should should be examined for the presence of papilledema. Papilledema will confirm your suspicion that she has transtentorial herniation, in which brain tissue bulges out of the cranium through the tentorial notch because of the increased intracranial pressure. You should schedule Magnetic Resonance Imaging (MRI) of the endocranium. While waiting for the MRI, undertaking a simple fundoscopic examination in the office will contribute to your diagnosis. Lumbar puncture is contraindicated in focal lesions, particularly when you suspect transtrentorial herniation, because by taking a sample of cerebrospinal fluid, brain tissue can press vital centers in the brainstem and cause the death of the patient. Doll's eye maneuver is a test of brainstem integrity and is performed in comatose patients; when the patient's head is quickly moved to one side and then to the other and if the brainstem is intact, the eyes will move conjugately away from the direction of turning and revert back to the midline. Having the history of rheumatoid arthritis and a stiff neck, your patient may have atlantoaxial subluxation, and moving the neck can cause serious damage to the cord. Your patient might have metastases in the lungs and you may think to obtain chest X-ray. However, her symptoms are symptoms of intracranial hypertension, not a process in the lungs.

Case A woman presents with an otherwise healthy 12-month-old boy because she noticed that his eyes that appear to be crossed as well as each one being different colors. The mother sheepishly admits to smoking for about 10 years and she smoked during her entire pregnancy. She is worried because she had a brother die when he was 3 because he had a tumor in both his eyes. Physical exam reveals heterochromia iridis and leukocoria bilaterally, as well as evidence of severe strabismus. Question What is the main risk factor for the most likely diagnosis of this child?

Correct answer: Heredity Explanation The correct response is heredity. The child most likely has inherited retinoblastoma. Retinoblastoma is a rapidly developing cancer that generally affects children under 6. It is most commonly diagnosed in children 1-2 years old. Genetic counseling is especially important when more than one family member has had the disease or if the retinoblastoma occurs in both eyes. Leukocoria is seen as a whitish color behind the pupil, which is usually black. It is a sensitive test best done by looking at the "red reflex." Normally, red reflection occurs in people's eyes when taking flash photographs. Dimming the room lights and using a flashlight to shine light directly into the child's eyes can also elicit the red reflex. With leukocoria, also known as "cat's eye," red reflex is absent. This abnormality is present in approximately 60% of all children with retinoblastoma. Keeping in mind that retinoblastoma is the third most common cancer overall affecting children, red reflex is a useful screening tool. It is a sign and not a risk factor for the disease. Although smoking can affect pregnancy and a child, in this particular case, heredity is probably the main risk factor. Heterochromia iridis is a relatively late symptom caused by the tumor invasion and/or neovascularization. It is a symptom, not a risk factor. Crossed eyes, or strabismus, which occurs as a result of visual loss, is a common sign of retinoblastoma. That is the reason funduscopic examination through a well-dilated pupil must be performed in all cases of childhood strabismus. Strabismus is usually secondary to macular involvement. It is also a sign and not a risk factor for the disease.

Case A 43-year-old woman presents to a walk-in clinic with a complaint of a painful, red lump on her left eyelid. Examination shows an oozing erythematous pustule surrounded by hyperemia edema. Question What is the most likely diagnosis?

Correct answer: Hordeolum Explanation Hordeolum is an inflamed sebaceous gland near a hair follicle which appears as a red, painful pustule and is sometimes known as a sty. Hordeola are usually self-limited, spontaneously improving in 1-2 weeks. Medical therapy for hordeola includes eyelid hygiene, warm compresses and massages of the lesions for 10 minutes 4 times per day, and topical antibiotic ointment in the inferior fornix if the lesion is draining. Chalazion is a granuloma of a meibomian gland and often causes lid protrusion. Pterygium is an extension of the pinguecula over the limbus toward the center of the cornea. It is neither painful nor red. Interstitial keratitis is a cloudy, painful, cornea caused most commonly by syphilis, though it has also been associated with tuberculosis. Staphyloma is protrusion of the sclera near the cornea resulting from injury to the sclera or increased intraocular pressure.

Case An 8-year-old child is brought to your office because of swelling of the left upper eyelid; the swelling is associated with redness and tolerable pain. No fever is noted. Physical examination shows a localized swelling and redness on the upper middle lid of the left eye; there is slight tenderness on palpation. Vital signs are within normal limits. Question Ico-delete Highlights What is the most likely diagnosis?

Correct answer: Hordeolum Explanation The clinical picture is suggestive of hordeolum, which is an infection of the lid glands. The most common causative agent is Staphylococcus aureus, which may either be acute or subacute. When the meibomian glands are infected it is referred to as internal hordeolum, wherein the lesion tends to be large and extend to the skin or conjunctival surface. If it affects the glands of Zeis and Moll, it is referred to as an external hordeolum or stye. It is smaller, more superficial, and points to the lid margins. Treatment, like any abscess, is warm compresses and surgical drainage (if needed). Topical antibiotics may also sometimes be used. If left untreated, it may progress to cellulitis of the lid or orbit, which requires systemic antibiotics. Recurrence is also frequent, and children with recurrent styes should be evaluated for an immunologic problem. Blepharitis is an inflammation of the lid margins characterized by redness and a scaling or crusting lesion. It is initially manifested by itching, irritation, and burning sensation. It is recurrent, chronic, and usually bilateral. In cases of the seborrheic type, the scales are greasy, erythema is less, and ulceration seldom occurs. In cases of the staphylococcal type, ulceration is common; lashes may fall out, and it is often accompanied by conjunctivitis and superficial keratitis. Most of the blepharitis is of mixed type. Application of antistaphylococcal agent or sulfonamides directly to the lids daily is the treatment of choice. Daily cleaning of the lid with a moist cotton applicator to remove scales and crusts is very helpful. Chalazion is an inflammation of the meibomian glands characterized by a firm, non-tender nodule on the upper eyelid. It differs from hordeolum because it does not have the presence of inflammatory signs. Excision is recommended if the nodule is large enough to cause astigmatism by exerting pressure on the globe. Some cases subside spontaneously. Entropion is a condition in which the lid margin is directed inwards. It usually causes discomfort and corneal damage because the eyelashes are also turned inwards. It is most commonly caused by scarring due to inflammation seen in trachoma; it may also result from Steven-Johnson syndrome. Surgery is effective. Ectropion is the opposite of entropion, in which the lid margin is turned outwards or everted; it is associated with an overflow of tears, maceration of the lid skin, inflammation of exposed conjunctiva, and/or superficial exposure keratopathy. Scarring from inflammation, burns, trauma, or orbicularis muscle weakness from facial palsy are the common causes. Surgical correction is necessary to protect the cornea.

Case A 35-year-old woman presents with a painful swelling of her left eyelid. On physical exam, there is tenderness to palpation and erythematous swelling present on the lid margin involving the eyelashes. Question What is the most likely diagnosis?

Correct answer: Hordeolum Explanation The correct answer is hordeolum (or stye), which is an infection that occurs at the lid margin. It is most often caused by bacteria and is treated with warm compresses and sometimes antibiotic ointment. An internal hordeolum is an infection of the meibomian gland that expands onto the lid conjunctiva. An external hordeolum is typically smaller than an internal one and found on the lid margin. Xanthelasma is a raised yellowish plaque in the skin of the eyelids. It is not painful and is classically associated with high cholesterol. A chalazion is a chronic inflammation of the meibomian gland inside the eyelid, not the lid margin, and is characteristically hard and nontender. Pinguecula is a nodule in the bulbar conjunctiva. Dacryocystitis is an inflammation of the lacrimal sac that leads to pain, swelling, and erythema around the tear sac.

Case A 17-year-old baseball player presents to the clinic after being struck in the eye with a baseball. On examination, you note bright red blood in the anterior chamber. Question What is your initial diagnosis?

Correct answer: Hyphema Explanation The clinical picture is suggestive of a hyphema. Hyphema is defined as hemorrhage into the anterior chamber. Patients with a corneal abrasion note severe pain and photophobia following a history of a traumatic event to the affected eye. Pinguecula is a yellow, elevated conjunctival nodule; it is commonly located on the nasal side of the eye. Most retinal detachments occur spontaneously, and blood in the anterior chamber is not found. Hypopyon is described as pus (white and cloudy fluid) in the anterior chamber; it usually follows a fungal infection.

Case A 70-year-old man presents with paralytic strabismus with maximal esotropia as he gazes to the left. Question What nerve is most likely affected in this case?

Correct answer: Left 6th cranial nerve Explanation The correct answer is the left 6th cranial nerve. This nerve abducts the eyeball and paralysis of this nerve results in the inability of the left eye to gaze laterally, causing maximal esotropia with left gaze. The right 6th cranial nerve will cause the same symptoms but as the patient gazes to the right. Cranial nerves III and IV are responsible for other movements of the eyeball, such as the ability to gaze upward and downward as well as pupil reaction to light and accomodation, which will not result in the defects noted in this patient.

Question Ico-delete Highlights A 44-year-old man presents for follow-up of poorly controlled type I diabetes mellitus, which was diagnosed 32 years ago. What change on his funduscopic examination would indicate a need for urgent referral to an ophthalmologist?

Correct answer: Neovascularization Explanation Neovascularization is the hallmark of proliferative diabetic retinopathy. New vessels can appear at the optic nerve and the macula as a result of retinal hypoxia. They are susceptible to rupture, resulting in vitreous hemorrhage, retinal detachment, and blindness. Proliferative retinopathy requires urgent referral to an ophthalmologist and is usually treated with pan retinal laser photocoagulation. The risk of developing diabetic retinopathy is related to the extent of glycemic control and the duration of diabetes. It is classified as nonproliferative and proliferative. Blot hemorrhages, cotton wool spots, and microaneurysms are indicative of nonproliferative diabetic retinopathy, which is usually seen 10 to 20 years after the onset of diabetes. Nonproliferative retinopathy does not always progress to proliferative retinopathy, but if it becomes extensive, it can result in retinal ischemia, which increases the likelihood of proliferative disease. Flame-shaped hemorrhages are indicative of hypertensive retinopathy.

Question Ico-delete Highlights A 36-year-old woman presents with a small and irregular right pupil. On exam, you note that the pupil does not respond to direct or consensual light stimuli; however, it becomes smaller during an accommodation testing. What is the most likely diagnosis?

Correct answer: Neurosyphilis Explanation The clinical picture is suggestive of neurosyphilis; more specifically, it is likely tabes dorsalis. The pupil described here is the Argyll Robertson pupil. The pupil reacts poorly to light, but it reacts well to accommodation. Signs and symptoms seen in a TIA include temporary weakness and heaviness of the contralateral arm, leg, or face. There may be monocular vision loss in the eye contralateral to the affected limbs, which are not described in this patient. Retinal vein or artery occlusion will produce sudden vision loss, which is not described in this patient. Herpes simplex can involve the eyes, but the patient would develop keratitis (corneal inflammation) with impaired vision, and dendritic ulcers can be seen with fluorescein stain.

Case A 64-year-old African-American man presents to the emergency department after he went blind in his right eye "out of the blue" 20 minutes ago. There is no pain associated with his symptoms and he is not nauseated. Past medical history is positive for DMII for the past ten years. The pupil reaction on the left side is normal with pressure of 17mmHg. Right pupil evaluation reveals no reaction to light or accommodation with pressure of 20mmHg. Right eye ophthalmoscopy reveals arteriolar narrowing, vascular stasis, and "boxcar" pattern. Question What is the most likely diagnosis?

Correct answer: Occlusion of the central retinal artery Explanation The symptoms described above are typical for occlusion of the central retinal artery, which is a branch of the ophthalmic artery, in turn a branch of the internal carotid artery. The "boxcar" pattern is segmentation of the venous blood column, bilateral boxcar ring is a useful sign of circulatory arrest and death. Acute central artery occlusion is an emergency, since it results in permanent blindness if circulation is not restored within 30-60 minutes. An acute glaucoma attack is accompanied by severe pain with decreased vision. The patient usually reports seeing halos around light. The pupil is fixed in a mid-dilated position, and the eyeball is firm to pressure since the intraocular pressure is elevated. Subconjunctival hemorrhage onsets spontaneously and shows a painless, bright red patch on the sclera. It usually is caused by overexertion, is benign, self-limited and has no influence on the visus. Retinal detachment starts with the patient seeing dark, vitreous floaters, light flashes, and blurred vision, which progresses to blindness if not treated. Macular degeneration causes painless loss of visual acuity. There is altered pigmentation in the macula.

Case A 14-year-old boy is seen in the office for a sports physical for the freshman basketball team. Past history is significant only for a high degree of myopia bilaterally, first diagnosed at age 4 years, and a dislocated shoulder at age 10 years that was easily reduced. Grades are A's and B's. Family history is significant for several unidentified ancestors having died in their forties of an unidentified cardiovascular disorder. Physical examination reveals normal vital signs. Height is 6'1" and weight 145 lbs. The upper to lower segment ratio is 0.65 (decreased). Arm span is 76". The palate is highly arched. Mild pectus excavatum was present. A 2/6 early diastolic murmur is present and is best heard at the second intercostals space at the right sternal border. Arachnodactyly of the fingers and toes and generalized loose jointedness and pes planus are also present. Echocardiography reveals a tricuspid aortic valve with grade 1 out of 4 aortic regurgitation with a normal aortic root diameter. Question In addition to echocardiography, which evaluation would be most productive?

Correct answer: Ophthalmology evaluation Explanation Ectopia lentis, retinal detachment, and other ocular anomalies are frequent in Marfan syndrome. CNS anomalies are unusual in Marfan syndrome, although intracranial aneurysms are described. Growth hormone is not elevated in Marfan syndrome. In growth hormone excess, the body habitus is proportionate, as opposed to the disproportionately long limbs seen in this patient. No specific findings will be seen on muscle biopsy. Further, this patient is not weak. The presentation does not suggest a collagen vascular disorder warranting a rheumatoid factor

Question A 34-year-old woman presents to your office to establish care. Her past medical history is significant for gastritis. She has no other medical problems. As part of your new patient assessment, you perform a neurological examination. On confrontation with visual field testing, you note bilateral temporal field defects, specifically a bitemporal non-homonymous hemianopsia. The remainder of your neurological evaluation is unremarkable. What would be your next step in the management of this patient?

Correct answer: Order an outpatient MRI of the brain Explanation Bitemporal visual field loss localizes to the optic chiasm. In a 34-year-old patient, the most likely cause is a pituitary tumor. The next step in management would be to obtain brain imaging to verify the presence of a lesion and to evaluate its extent. An urgent referral to the emergency room is not indicated; at the patient's age, there is nothing to indicate a stroke. Thyroid function tests may be abnormal with a pituitary lesion. Evaluating thyroid hormone levels may be important in characterizing a pituitary lesion if one is present. The first step in management is to obtain brain imaging. Checking an EKG in the office is not indicated based on the information presented. While glaucoma can cause visual field defects, a bitemporal hemianopsia suggests a chiasmal lesion. Glaucoma would be unlikely in this setting and with a patient of this age.

Case A 32-year-old woman with a past medical history of multiple environmental allergies presents with bilateral ocular pruritis, eye redness, and excessive tearing over the last several days. She notes associated nasal congestion. She states that these symptoms are predictable, occurring at approximately the same time every year. She has had these symptoms for many years, and over-the-counter medications are not helpful. She denies ocular trauma, fever, chills, swollen glands, otalgia, otorrhea, headache, and sore throat. She further denies shortness of breath, cough, and chest pain. Her physical exam is remarkable for bilateral conjunctival erythema and chemosis, violaceous mucosal turbinates in the nasal cavity, and small nasal polyps. Question Ico-delete Highlights What health maintenance strategy is correct regarding this patient?

Correct answer: Pillow and mattress covers, air purifiers, and dust filters should be purchased. Explanation This patient's diagnosis is allergic rhinitis and conjunctivitis. Maintaining an allergen-free environment by covering pillows and mattresses with plastic covers, substituting synthetic materials for animal products, removing dust-collecting household fixtures, and using air purifiers and dust filters are all recommended. Warm compresses are useful in managing hordeolums and blepharitis, not allergic conjunctivitis. Numerous over-the-counter antihistamines offer the benefit of reduced cost but are associated with higher rates of drowsiness. When using intranasal corticosteroids, patients should be reminded that there may be a delay in the onset of relief for 2 or more weeks. Corticosteroid sprays may also shrink nasal polyps, delaying or eliminating the indications for endoscopic sinus surgery.

Case A 30-year-old woman presents to the emergency room at 7 am with severe pain and swelling of her right eye. She was awakened early the previous evening due to the discomfort and swelling of the surrounding conjunctiva. She found it difficult to sleep due to the discomfort. She planned on going to work, but the swelling had closed her eye shut, and she developed excruciating pain in the eye that radiated internally. The patient denies recent swimming and does not recollect any previous trauma or injury to the eye. She uses contact lenses, but they were not in use due to the condition of her eye. The contact lenses were stored in a small pillbox container with some fluid that she later described as tap water. She ran out of sterile cleaning and soaking solution for the contact lenses, so she has been using tap water as a substitute for approximately 5 days. She frequently sleeps with her contacts in. The patient is afebrile. Pulse is 70/min, and blood pressure is 135/80 mm Hg. Lungs are clear, and there is no evidence of lymphadenopathy. The eye has profound conjunctivitis that is acute and follicular. Purulent drainage is present. The acute nature of conjunctivitis requires an ophthalmologist consult. The ophthalmologist obtains ocular fluid for culture and Gram stain. CBC results are unremarkable. The Gram stain reveals the following results (see image). Prompt and aggressive therapy is initiated. Question Ico-delete Highlights What is the most likely organism causing this acute eye threatening infection?

Correct answer: Pseudomonas aeruginosa Explanation Pseudomonas aeruginosa causes a supprative keratitis that is rapid in its development. It can be caused by previous injury to the eye, or possible, recurrent, minor corneal epithelial trauma, caused by contact lenses. The incidence is higher with soft lenses and extended overnight wear. Fever is usually absent, and leukocytosis is absent or minimal. The infection can lead to corneal ulceration, resulting in the rapid loss of ocular function; therefore, these infections need to be approached as a medical emergency. It is a very common opportunistic source of human infections, especially in the hospital setting. Pathogenesis is due to its minimal nutritional requirements, relative resistance to antibiotics, and a host of other invasive and toxicogenic substances that it produces. Scrapings from the floor of the ulcer exhibiting Gram-negative rods are strongly indicative of Pseudomonas aeruginosa and should necessitate treatment. Immediate initiation of combined topical and subconjunctival therapy with an animoglycoside antibiotic such as gentamicin or tobramycin is advised. Topical steroids are sometimes used to reduce ocular inflammation. Pseudomonas aeruginosa is a Gram-negative rod; it is a non-lactose fermenting, oxidase-positive motible bacteria. Growth on MacConkey agar is usually characterized by the production of a "grape-like" smell. A blue-green color, due to the production of the diffusible fluorescent pigments pyoverdin and pyocyanin, is characteristic of the colonies growing on MacConkey. Haemophilus aegyptius is indigenous to humans. It is an important cause of purulent conjunctivitis called "pink eye", and it can occur in outbreaks because of its contagious nature. The infection is not acute in presentation. The diffuse pink color of the sclera and the presence of a serous or purulent discharge are virtually diagnostic of Haemophilus aegyptius infection. Leukocytosis is absent. The infection is not acute in presentation. The treatment of Haemophilus aegyptius is with topical antibiotics. Because of the infectious nature of the infection, instructions should be provided to the patient to help prevent the spread of the infection to others. The organism is a Gram-negative coccobacillus; it is non-motile, fastidious bacteria requiring the presence of special factors for its growth on agar media. These factors are hemin and nicotinamide adenine dinucleotide, which are present in chocolate agar. Acanthamoeba is a corneal infection that occurs in healthy people and is often associated with contact lens wearers. Swimming in fresh water (where the organism is naturally found) with contact lenses can predispose the wearer to Acanthamoeba keratitis. To prevent infection, it is recommended that contact lenses be cleaned and stored with Benzalkonium chloride-preserved saline and solutions containing thimerosal with edetate. The keratitis is slow in developing and is often mistaken for herpes, bacterial, or fungal keratitis. The average delay to definitive treatment can range from days to months. Symptoms include blurred vision, conjunctivitis, tearing, severe pain to the eye, and photophobia. The keratitis achieves an advanced stage in several days to several months, and it can exhibit patchy stromal infiltrates and dendriform epithelial involvement without frank corneal ulceration in its early stages. A ring corneal infiltrate is characteristic of this keratitis in its late stages. Early diagnosis, aggressive surgical debridement, and medical management can prevent eye damage. High concentrations of topical antimicrobial drugs (1% miconazole, 0.1% propamidine isethionate, and Neosporin) for a minimum of 3 - 4 weeks is part of the antibiotic therapeutic regimen employed in the treatment of Acanthamoeba keratitis. Acanthamoeba keratitis is a free-living amebae that can cause granulomatous amebic encephalitis and keratitis. Detection is usually made by observing the free-living motile organisms in a wet prep preparation. Staphylococcus aureus is probably the 2nd most common bacterial isolate of human infections behind Escherichia coli and the most common cause of bacterial endophthalmitis. Conjunctivitis caused by Staphylococcus aureus is usually characterized as non-severe where there is little to no lid edema, scant purulent discharge, and normal cornea; however, in some cases, the presentation can be severe. Topical agents are usually used to treat this infection such as cephalosporins or semisynthetic penicillins. In suspected case of resistance, topical vancomycin should be considered. Staphylococcus aureus has a host of invasive and toxigenic characteristics that enhance the pathogenesis of the organism in the human host. Beside causing bacterial endophthalmitis, the organism has been described as an etiologic agent of many infections including but not limited to, endocarditis, septicemia, abscesses, and urinary tract infections. It is a Gram-positive staining cocci that is catalase positive and coagulase positive. Candida albicans is a yeast. Endophthalmitis due to yeast is generally a common and serious complication of intravenous drug use. Candida albicans is the most common fungal cause. It is usually of hematogenous origin, where the patient has infective endocarditis or some other infective process occurring. The symptoms are blurred vision, decreased vision, white cotton appearing exudative lesions in the choroid and retina with vitreous haziness, and eye pain. A definitive diagnosis is made by obtaining vitreous fluid for Gram stain and culture. The treatment consists of parenteral amphotericin B together with flucytosine. Intraocular amphotericin B administration as therapy is controversial. The incidence of permanent intraocular damage is high. Yeast appears on Gram stain as large Gram-positive organisms that are approximately 3 - 5 times larger than Gram-positive cocci. They are aerobic and generally grow well on most non-selective agar media.

Case A 68-year-old woman presents with episodic, monocular blindness lasting typically less than 5 minutes described as a curtain moving vertically over her visual field. She denies pain or other related vision symptoms. She is asymptomatic at the time that she presents to your office, and her fundoscopic exam reveals no significant abnormality. Question What is the most likely cause of the condition described?

Correct answer: Retinal artery emboli Explanation The answer is retinal artery emboli, as the diagnosis for this patient is amaurosis fugax. Amaurosis fugax is characterized by brief episodes of monocular blindness caused by retinal artery emboli, often from ipsilateral carotid disease. The majority of patients will be asymptomatic at the time that they are examined, and therefore their retinal exam may well be normal (the classically described findings of this condition, such as papilledema, cherry-red spot, and retinal pallor, are generally seen only during or immediately after an active episode). The other answers are not correct. A detached retina does not cause brief, episodic occurrences of blindness, and retinal detachment would be noted on fundoscopic exam. Retinal vein occlusion and macular degeneration do not cause the symptoms noted above, and fundoscopic exam would reveal significant abnormalities. Papilledema would typically cause bilateral episodic blindness, and fundoscopic exam would demonstrate optic disc swelling.

Case A 2-week-old female infant is seen for her newborn well-baby exam after a normal birth and delivery. She has been nursing well, has regained her birth weight and her development appears normal for her age. Physical examination is normal with the exception that ophthalmoscopic evaluation reveals a faint white reflex in her right eye. Question What is the most likely diagnosis?

Correct answer: Retinoblastoma Explanation Retinoblastoma is the most common primary malignant intraocular tumor of childhood. It usually appears quite early in the first 5 years. Leukocoria, a white or Cat's eye reflex in the pupil is the most frequent finding. There may also be strabismus due to vision impairment. Ocular inflammation, intraocular hemorrhage, glaucoma, or heterochromia iridis may be seen. On fundoscopic exam, the tumor may appear as a small to large white mass depending on its stage. Primary treatment includes enucleation, though smaller tumors diagnosed at an earlier stage may be amenable to newer alternative treatments such as cryotherapy and photocoagulation. Though leukocoria may be seen in retrolental fibroplasia or advanced stage of retinopathy of prematurity, it is predominantly a disorder in preterm, low birth weight infants who received supplemental oxygen in the newborn period. These infants are susceptible due to the immaturity and subsequent damage of developing retinal vasculature. If the retina goes through various stages to ischemia and neovascularization, leukocoria may be seen representing retinal detachment and a subsequent membrane formation. Phakomata are retinal findings hallmarking hamartomatous disorders such as tuberous sclerosis. The distinctive ocular lesion is a yellowish multinodular cystic lesion arising from the retina or disc. Similar lesions can occur in neurofibromatosis. Retinitis pigmentosa is a progressive degeneration of the retina characterized by pigmentary changes, arteriolar attenuation, some degree of optic atrophy and progressively deteriorating visual impairment. Granularity or mottling of the retinal pigment pattern or distinctive focal pigment aggregates can be seen via fundoscopy. Retinoschisis is a congenital disorder involving splitting of the retina into an inner and outer layer. Usually, good vision is maintained. An elevation of the inner layer of the retina can be seen.

Case A 48-year-old Caucasian man presents with acute onset of blurring of vision and severe pain in the left eye that began 30 minutes ago. He notes seeing halos with his left eye; he is also experiencing nausea and vomiting; these symptoms started at the same time as the pain. The patient reports that he was relaxing on his porch when the pain started. His temperature is 36.9°C, pulse 90/min, BP 130/90 mm Hg, and respirations 20/min. Physical examination reveals a shallow anterior chamber, a hazy cornea, a fixed, moderately dilated pupil, and ciliary injection. Question What would be the next step in the management of this patient?

Correct answer: Tonometry Explanation The correct response is tonometry. The history and physical examination of this patient are suggestive of acute angle-closure glaucoma, which can be easily confirmed by measuring the intraocular pressure using a tonometer (e.g., the Schiötz tonometer). Acute angle-closure glaucoma develops in individuals with pre-existing anatomic narrowing of the anterior chamber, which is seen mainly in far-sighted people. The condition usually develops in the twilight hours, which is when the pupil is dilated in response to the low level of illumination. It may also occur with pupillary dilation for ophthalmoscopy, so topical atropine would be contraindicated. Acute angle-closure glaucoma is an ophthalmologic emergency. Treatment involves immediate lowering of the intraocular pressure via systemic acetazolamide to decrease the production of aqueous humor, which should be supplemented with topical hyperosmotic agents and topical beta-blockers. Topical pilocarpine is then used to cause miosis. Once the intraocular pressure is under control, a peripheral iridectomy can be done to prevent against future recurrences.

Case A 25-year-old man presents with a 1-day history of redness, itching, and swelling in the right upper eyelid. The redness has increased; there has been no eye discharge or watering. On examination, a pustular swelling at the root of the lashes is found. It is surrounded by redness, and it is tender on palpation. The lesion is shown in the image. Question What is the appropriate management of this patient?

Correct answer: Warm compresses and local antibiotic cream Explanation There are 2 types of hordeolum. The first, external hordeolum, is an acutely inflamed stye is caused by infection of gland of Zeis (sebaceous glands near the eyelashes) or Moll (apocrine glands of the lid margin). It causes redness, irritation, itching, and then a white dot surrounded by erythema appears at the lash root. Warm compresses plus erythromycin cream is usually effective in most cases. The other type is internal hordeolum caused by lipogranulomatous inflammation of the meibomian gland of the tarsus. It is caused by obstruction of the gland orifice and causes a lid swelling that typically diminishes in size with voluntary squeezing of the eyelid. It may be acutely inflamed and causes diffuse lid swelling and erythema. Commonly there is remission and exacerbation. The treatment of the acute form is systemic antibiotics, hot fomentation, and topical erythromycin cream, while chronic persistent form usually requires surgical removal and curettage from the conjunctival side. Blepharitis is inflammation of the lid margin either due to allergy or infection. It is mostly associated with dry secretions, scales, redness, and there may be itching. Infective cases are treated with antibiotic drops, and allergic blepharitis is usually treated with local steroids. Conjunctivitis is inflammation of the conjunctiva and is always associated with red eye and copious eye discharge. Topical antibiotics are used to treat infective conjunctivitis, while steroids and mast cell stabilizers are used to treat allergic types. Herpes zoster ophthalmicus is an infection due to herpes zoster affecting the ophthalmic division of trigeminal nerve. It causes rash on the upper eyelid, which is extremely painful, unilateral, and may affect the cornea, causing keratitis that presents with photophobia, lacrimation, and severe pain. It is typically treated with topical antivirals and steroids. Systemic antivirals are sometimes used in this case. Antifungal cream is not useful in this patient, as external hordeolum is a bacterial infection.


Conjuntos de estudio relacionados

Ivy Tech PHYS 101 Summer Midterm Study Guide

View Set

Principals of cybersecurity Chap 7-12

View Set

Spanish 2 - Lección 6 - Recapitulación

View Set